f17sugg

70
MANAGEMENT ACCOUNTING Suggested Answers Final Examination –Summer 2013 Page 1 of 6 A.1 Effect of change in debt collection policy Rs. in million Existing sale 15,575 Existing credit sale (80% of 15,575) 12,460 Existing Debtors 2,590 Debtors turnover (2,590 × 360 ÷ 12,460) 75 days Conversion costs (13,770 × 50 ÷ 150) 4,590 Fixed cost (20% of 4,590) 918 Contribution margin (%) [(1805 + 918) ÷ 15,575] 17.48% New credit sale as per proposal (92% of 12,460) 11,463 Decrease in credit sales (12,460 11,463) 997 Loss of contribution margin on decrease in sale (17.48% × 997) (174.28) Reduction in bad debts [(4% × 12,460) (3% × 11,463)] 154.51 Early payment discount to debtors (11,463 × 40% × 1%) (45.85) Surcharge from late payment [2866 (W-1) × 24 × 45 ÷ 1,000] 30.95 Savings from decrease in debtors [(2,590 1,337) × 15%] 187.95 153.28 Revising the credit policy is feasible W-1: Debtors analysis Credit sale Debtors Debtors turnover Debtors availing discount 4,585 382 30 days Debtors paying within 40 days 4,012 446 40 days Other debtors 2,866 509 64 days 11,463 1,337 42 days Effect of change in creditors payment policy Creditors turnover in days as given in the question 40.00 Cost of materials (13,770 × 2 ÷ 3) for the year 9,180 Creditors outstanding (9,180 × 40 ÷ 360) 1,020 Revised creditors with turnover of 60 days (1,020 × 60 ÷ 40) 1,530.00 Increase in creditors / release of funds (1,530 1020) 510.00 Saving from funds released (15% × 510) 76.50 Discount lost due to delayed payment to creditors (9,180 × 1%) 91.80 Loss due to delayed payment to creditors (15.30) Delayed payments to creditors is not feasible - http://gcaofficial.org

Upload: ahmed-raza-mir

Post on 14-Apr-2017

218 views

Category:

Documents


2 download

TRANSCRIPT

MANAGEMENTACCOUNTINGSuggested Answers 

    Final Examination –Summer 2013 �

                                                                                    Page 1 of 6   

A.1 Effect of change in debt collection policy Rs. in million Existing sale 15,575 Existing credit sale (80% of 15,575) 12,460 Existing Debtors 2,590 Debtors turnover (2,590 × 360 ÷ 12,460) 75 days Conversion costs (13,770 × 50 ÷ 150) 4,590 Fixed cost (20% of 4,590) 918 Contribution margin (%) [(1805 + 918) ÷ 15,575] 17.48% New credit sale as per proposal (92% of 12,460) 11,463 Decrease in credit sales (12,460 – 11,463) 997

Loss of contribution margin on decrease in sale (17.48% × 997) (174.28) Reduction in bad debts [(4% × 12,460) – (3% × 11,463)] 154.51 Early payment discount to debtors (11,463 × 40% × 1%) (45.85) Surcharge from late payment [2866 (W-1) × 24 × 45 ÷ 1,000] 30.95 Savings from decrease in debtors [(2,590 – 1,337) × 15%] 187.95

153.28 Revising the credit policy is feasible W-1:

Debtors analysis Credit sale Debtors Debtors turnover

Debtors availing discount 4,585 382 30 days Debtors paying within 40 days 4,012 446 40 days Other debtors 2,866 509 64 days

11,463 1,337 42 days Effect of change in creditors payment policy Creditors turnover in days as given in the question 40.00 Cost of materials (13,770 × 2 ÷ 3) for the year 9,180 Creditors outstanding (9,180 × 40 ÷ 360) 1,020 Revised creditors with turnover of 60 days (1,020 × 60 ÷ 40) 1,530.00 Increase in creditors / release of funds (1,530 – 1020) 510.00 Saving from funds released (15% × 510) 76.50 Discount lost due to delayed payment to creditors (9,180 × 1%) 91.80 Loss due to delayed payment to creditors (15.30) Delayed payments to creditors is not feasible -

http://gcaofficial.org

MANAGEMENTACCOUNTINGSuggested Answers 

    Final Examination –Summer 2013 �

                                                                                    Page 2 of 6   

A.2 Job Costing Order quantity 2,000,000 Finished running meters (0.04 × 2,000,000) 80,000 Lamination film input in meters (80,000 ÷ 98%) 81,633 Printing film input in meters (81,633 ÷ 97%) 84,158 Set-up consumption 1,200 85,358

Meters Kg Rate Amount in Rs. Raw materials Printing film 85,358 * 2,439 260 634,140 Lamination film 81,633 **2,332 260 606,320 Ink (2 × 84,158 ÷ 1,000) 168 180 30,240 Chemical (4 × 84,158 ÷1,000) 337 150 50,550 Glue (81,633 ÷ 250) 327 110 35,970 1,357,220 Labour cost Skilled labour [250 ÷ (200 × 90%) × 35,000] 48,611 Unskilled labour [750 ÷ (200 × 85%) × 15,000] 66,176 114,788 Overheads Printing overhead (85,358 × 5) 426,790 Lamination overhead (81,633 × 3) 244,899 671,689 Total cost 2,143,697 Profit margin (25% on cost) 535,924 Selling price 2,679,621

*85,358 ÷ 35 = 2,439 **81,633 ÷ 35 = 2,332 A.3 --------------------------------------Amount in thousands-------------------------------------

Scenarios 1 2 3 4 5 6 Demand 7,000 7,000 8,000 8,000 9,000 9,000 CM per unit 12.00 11.00 12.00 11.00 12.00 11.00 Contribution Margin 84,000 77,000 96,000 88,000 108,000 99,000 Fixed cost other than dep. (15,000) (15,000) (15,000) (15,000) (15,000) (15,000) Depreciation (300 – 30)÷15 (18,000) (18,000) (18,000) (18,000) (18,000) (18,000) Financial charges (W-1) (25,200) (25,200) (25,668) (25,668) (26,064) (26,064) Net profit 25,800 18,800 37,332 29,332 48,936 39,936

Equity 140,000 140,000 142,600 142,600 144,800 144,800 Rate of return-% 18.43 13.43 26.180 20.570 33.800 27.580 Probability (Demand) 0.50 0.50 0.30 0.30 0.20 0.20 Probability (CM / unit) 0.35 0.65 0.35 0.65 0.35 0.65 Probability of scenario 0.175 0.325 0.105 0.195 0.070 0.130 Estimated rate of return-% 3.23 4.36 2.749 4.011 2.366 3.585

Probability for getting required rate of return of 20% 0.50

Rate of return based on all the possible scenarios - % cumulative 20.30

W-1: Financial charges Capital investment 300,000 300,000 300,000 300,000 300,000 300,000 Working capital 50,000 50,000 56,500 56,500 62,000 62,000 350,000 350,000 356,500 356,500 362,000 362,000 Equity 40% 140,000 140,000 142,600 142,600 144,800 144,800 Bank borrowing 210,000 210,000 213,900 213,900 217,200 217,200 Finance charges @ 12% 25,200 25,200 25,668 25,668 26,064 26,064

MANAGEMENTACCOUNTINGSuggested Answers 

    Final Examination –Summer 2013 �

                                                                                    Page 3 of 6   

A.4 (a) Fixed cost of B: Rupees Decrease in variable and fixed costs (2,500,000 – 1,800,000) 700,000

Decrease in units of product B (10,000 – 2,000) 8,000 Variable cost per unit (700,000 ÷ 8,000) 87.50 VC for 2,000 units @ 87.50 per unit 175,000 Fixed cost of B (1,800,000 – 175,000 ) 1,625,000 Contribution margin (CM) per unit of B (based on 2013 results) Sales 750.00 Raw material (300 + 15%) 345.00 Direct wages 200.00 Variable overheads 87.50 632.50 CM per unit of B 117.50 Shut down point: Avoidable FC / CM per unit Avoidable FC (1,625,000 – 162,500 – 450,000) 1,012,500 Units to breakeven i.e. shut down point (1012500 ÷ 117.50) 8,617 (b) Contribution Margin (CM) per unit of A :

Sales 2000 Raw material (500 + 15%) 575 Direct wages 375 Variable overheads W-1 50 1000 1000 Net profit of A for 31.03.13 (20 – 5 – 3.75 – 3) 8,250,000 Net loss of B for 31.03.13 (1.5 – 6 – 4 – 1.8) (1,300,000) Profit for 2013 6,950,000 Increase in profit required (20% of 6,950,000) 1,390,000 Fixed cost – A W-1 2,750,000 Fixed cost – B (162,500 + 450,000) (plant is shut down) 612,500 Variable cost of 13,000 units at Rs. 1,000 per unit 13,000,000 Contribution Required 24,702,500 No. of units to be produced 13,000 Price per unit of A to be charged (24,702,500 ÷ 13,000) 1900 W-1: Fixed cost of A 2014 2013 Total variable and fixed cost of 13,000 and 10,000 units of A 3,400,000 3,000,000 Increase in fixed cost during 2014 250,000 3,150,000 3,000,000 Variable cost of 3,000 units 150,000 Variable cost per unit (150,000 ÷ 3,000) 50 VC at 13,000 and 10,000 units 650,000 500,000 Fixed cost of A 2,750,000 2,500,000

MANAGEMENTACCOUNTINGSuggested Answers 

    Final Examination –Summer 2013 �

                                                                                    Page 4 of 6   

A.5

Activity

Expected Time

ES EF LS LF Float

1 – 2 5 0 5 0 5 0 1 – 3 9 0 9 6 15 6 1 – 4 8 0 8 6 14 6 2 – 4 6 5 11 8 14 3 2 – 5 5 5 10 5 10 0 3 – 6 9 9 18 15 24 6 4 – 6 10 11 21 14 24 3 5 – 7 10 10 20 10 20 0 6 – 8 7 21 28 24 31 3 7 – 8 11 20 31 20 31 0

S. No. Path Duration in hours

(i) 1 - 2 - 5 - 7 - 8 31 (ii) 1 - 2- 4 - 6 - 8 28 (iii) 1 - 3 - 6 - 8 25 (iv) 1 - 4 - 6 - 8 25

Therefore, the critical path is path (i)

MANAGEMENTACCOUNTINGSuggested Answers 

    Final Examination –Summer 2013 �

                                                                                    Page 5 of 6   

A.6 (a) (i) Net profit for the year South East Sales (24,000 × 900), (26,400 × 1,100) 21,600,000 29,040,000 VC (24,000 × 700), (26,400 × 775) (16,800,000) (20,460,000) Contribution margin for the year 4,800,000 8,580,000 FC (24,000 × 100), (26,400 × 150) (2,400,000) (3,960,000) Net profit for the year 2,400,000 4,620,000 Residual Income for the year Net profit 2,400,000 4,620,000 Required return on assets [15% × (11.10)] [15% × (25.8)] 1,665,000 3,870,000 Residual Income for the year 735,000 750,000 Return on investment (ROI) Net profit 2,400,000 4,620,000 Average operating assets 11,100,000 25,800,000 ROI % [2,400,000 ÷ 11,100,000] [25,800,000 ÷ 4,620,000] 21.62% 17.91% RI reveals the same level of performance for each division. ROI indicates that division East is the underperforming division. However,

one reason could be that South has older plant and machinery and East has been established relatively recently and therefore the ROI of South is higher due to the fact that its plant may have been substantially written down due to depreciation.

To make a better comparison, a more appropriate base should be used. These may include fair value of assets or historical cost duly adjusted for rate of inflation. However, in case the base is changed, due recognition should be given to the fact that older machinery is less efficient and requires more maintenance etc.

(ii) Units that should be sold to achieve ROI as that of South division

Required return 21.62% Average operating assets 25,800,000 Amount of return 5,577,960 Fixed costs per annum 3,960,000 Total contribution required 9,537,960

Contribution margin per unit: Amount in Rs. Sale price 1,100 Variable costs (375 + 225 + 175) 775 Contribution margin per unit: 325

Units required to be sold per annum (9,537,960 ÷ 325) 29,348

(b) Additional contribution (12.5 × 8,580,000) 1,072,500 Depreciation (4,000,000 – 400,000) ÷ 5 (720,000) Revised total annual profit (4,620,000 + 1,072,500 – 720,000) 4,972,500 Operating investment (25,800,000 + 4,000,000) 29,800,000 ROI 16.69% Revised total annual contribution 4,972,500 Return on assets (15% × 29,800,000) 4,470,000 Residual income 502,500

MANAGEMENTACCOUNTINGSuggested Answers 

    Final Examination –Summer 2013 �

                                                                                    Page 6 of 6   

A.7 Rupees (a) Actual quantity at actual prices (given) 173,280 (b) Actual quantity in actual mix at standard prices

P 1,680 40 67,200 Q 1,650 30 49,500 R 870 60 52,200 168,900

(c) Actual quantity in standard mix at standard prices St. Mix St. Price

P 4,200 15÷35 1,800 40 72,000 Q 4,200 12÷35 1,440 30 43,200 R 4,200 8÷35 960 60 57,600 172,800 (i) Material price variance (a) - (b) 4,380 Adverse (ii) Material usage variance (1) Standard cost of actual output [3,648 × (1,440 ÷ 32)] 164,160 (2) Actual quantity at standard price 168,900 (3) Difference in above 4,740 Adverse

(iii) Mix variance (c) - (b) 3,900 Favourable (iv) Yield variance kg (1) Actual yield 3,648 (2) Standard yield for actual input

(4,200 × 32) ÷ 35

3,840 (3) Difference in above at standard cost

per unit of output [192 × (1,440 ÷ 32)] 8,640 Adverse

(THE END)

MANAGEMENT ACCOUNTING Suggested Answers

Final Examination - Winter 2012

Page 1 of 6

A.1 SGL Limited

Projected cash-flow statement for the year ending 31 December 2013

Rs. in million Inflows

Cash from customers W.1 1,328.36 Outflows

Payments for purchases W.3 (318.11) Payments for expenses W.4 (681.35) Payments for financial charges 16/4+(16*1.1*3/4) (17.20) Payments for dividend (80*20%)+(80*1.1*15%) (29.20)

(1,045.86)

Net inflows

282.50

Workings: W.1: Cash from customers: Gross sales for 2013 (W.2) 901.25*1.5 1,351.88 Cash sales net of 2% discount 1351.88*20% 270.38 Collection from trade debtors:

Trade debtors: Opening Balance

90.00 Credit sales for 2013 1351.88-(270.38/0.98) 1,075.98 Trade debtors: Closing Balance 90*1.2 (108.00)

1,057.98

1,328.36

W.2: Cost of sales for 2013 Raw material consumption 722*35%*1.2*1.08 327.50 Variable conversion cost 722*45%*1.2*1.1 428.87 Fixed conversion cost (722*20%-3)*1.06+3 152.88 Cost of goods manufactured

909.25

Opening finished goods inventory

89.00 Closing finished goods inventory

(97.00)

Cost of sales

901.25

W.3: Payments for purchases Raw material consumption W.2 327.50 Raw material - opening inventories (722*35%)*30/360 (21.06) Raw material - closing inventories 327.5*30/360 27.29

Total purchases for 2013

333.73 Trade creditors - opening balance

40.00

Trade creditors - closing balance 333.73*60/360 (55.62)

318.11

W.4: Payments for expenses

Variable and fixed conversion costs 428.87+152.88-3 578.75 Variable operating cost (100-9-16)*1.2*1.1 99.00 Fixed operating costs (advertisement) 16*1.06 16.96 Total costs for 2013 (excluding depreciation)

694.71

Payables for expenses – Opening Balance (722*65%-3+100-9)*35/360 54.18 Payable for expenses – Closing Balance 694.71*35/360 (67.54)

681.35

MANAGEMENT ACCOUNTING Suggested Answers

Final Examination - Winter 2012

Page 2 of 6

A.2 (a) Learning curve applications Learning curves may assist the management in the following areas. 1 Pricing decisions

Application of a learning technique may assist the management in determining the

cost of a project/order more accurately and thereby quoting the right price especially where bidding is expected to be highly competitive.

2 Work scheduling

Learning curves enable management to predict the labour requirement more

effectively and to prepare more accurate delivery schedules. 3 Standard setting

Learning curves may assist management in setting of more accurate budgets and

standards. (b) Cost of producing ordered bodies of washing machines

Material First 16 batches (16*66,000) 1,056,000 Next 84 batches 84*(66,000/1.1*1.06) 5,342,400

Direct labour cost Normal hours at Rs. 220 1,760,000 Overtime hours at Rs. 330 686,070

Overheads Normal hours at Rs. 150 1,200,000 Overtime hours at Rs. 187.5 389,813

Total costs Rs. 10,434,283 Order price at a margin of 25% of the selling price Rs. 13,912,377 W.1: Learning curve %:

Batch No. Cumulative hours

Average hours per batch

Learning curve %

1 200.00 200.00 2 (200+160) 360.00 180.00 (180/200) 90%

4 (360+148+140) 648.00 162.00 (162/180) 90% W.2: Overtime hours: Learning effect at 90% learning curve -0.152 Hours for first 64 batches 64*200*(64) 6,803 -0.152 Hours for first 63 batches 63*200*(63) ( 6,712) -0.152 Hours per batch after batch 91 Hours required:

First 64 batches 6,803 Last 36 batches (91×36) 3,276

Total hours 10,079 Overtime hours (10,079 – 8,000) 2,079

MANAGEMENT ACCOUNTING Suggested Answers

Final Examination - Winter 2012

Page 3 of 6

A.3 RCL

X Y Z Total

Production Units A 50,000 40,000 25,000

Cost allocation by activity:

1. Procurement department

Direct material cost per units at Rs. 200 per kg. B

400

300

500

Raw material consumption - kg B/200*A C 100,000 60,000 62,500

EOQ - kg

D 10,000 12,000 6,250

No. of purchase orders C/D 10 5 10 25

Procurement department cost

E 1,000 500 1,000 2,500

2. Batch set up cost

Batch size

F 500 250 250

No. of batches A/F G 100 160 100 360

Batch set-up costs

H 1,000 1,600 1,000 3,600

3. Quality control department

Inspection hours per batch

J 20 15 18

Inspection hours G*J K 2,000 2,400 1,800 6,200

Quality department costs

L 1,455 1,746 1,309 4,510

4. Other overheads

Direct labour cost per unit

M 300 350 250

Direct labour hour per unit M/50* N 6 7 5

Total direct labour hours A*N O 300,000 280,000 125,000 705,000

Utilities 1,800 1,680 750 4,230

Salaries of supervisors and foreman 1,500 1,400 625 3,525

Salaries of cleaners and maintenance staff 600 560 250 1,410

Miscellaneous expenses 300 280 125 705

P 4,200 3,920 1,750 9,870

Total costs Rs. (E+H+L+P) R 7,655 7,766 5,059 20,480

Costs per unit using ABC:

Direct material 400.00 300.00 500.00

Direct labour 300.00 350.00 250.00

Factory overheads R/A 153.10 194.15 202.36

Total

Rs. 853.10 844.15 952.36

MANAGEMENT ACCOUNTING Suggested Answers

Final Examination - Winter 2012

Page 4 of 6

A.4 Industrial Tools Limited

Relevant costs of producing Zee Material

cost Internal production of Beta (W.2) 7,520*757.50

5,696,400

External buying of Alpha (W.2) 13,984*1,100

15,382,400

Direct labour cost

W.4 1,120,000 Variable overheads (W.4) 20,600*120

2,472,000

Total relevant cost

Rs. 24,670,800 Zee selling price at 30% above the relevant costs 24,670,800*1.3 Rs. 32,072,040

W.1: Decision to produce Beta internally or not

Beta - internal production costs per kg - Variable 520+(50*1.25)+(120*1.25) 732.50 - Fixed (existing) (Not relevant) - - Fixed (additional) 188,000/9,400*1.25 25.00 757.50 Cost of Beta for each unit of Zee 757.5*2.5 1,894.00 Cost of material Alpha for each unit of Zee 1,100*2 2,200.00 Saving on producing Beta internally 306.00 Hence it is beneficial to produce Beta internally.

W.2: Internal production capacity for the substitute material Beta and buying of Alpha externally

Total hours available

30,000

Hours required for production of Zee

W.4 20,600

Capacity available for production of Beta

Hrs. 9,400

Beta production from the available capacity 9,400/1.25 Kg 7,520

Quantity of Alpha to be purchased externally (10,000-(7,520/2.5))*2 Kg 13,984

W.3: Variable overhead rate per labour hour for Zee and Beta

Variable overhead rate per labour hour 150-(900,000/30,000) 120.00

W.4: Direct Labour Cost for Zee

Units Hours per

unit Hours

Wages at higher of Rs. 100 per hour and Rs. 210 per unit

5,000 2.2 11,000 100 per hour 1,100,000

3,000 2.0 6,000 210 per unit 630,000

2,000 1.8 3,600 210 per unit 420,000

10,000

20,600

2,150,000

Payment of idle hours at 50% (non relevant cost) 20,600 *100/2 (1,030,000) Relevant labour cost for Zee 1,120,000

MANAGEMENT ACCOUNTING Suggested Answers

Final Examination - Winter 2012

Page 5 of 6

A.5 ICL

(a) Maximisation of profit using revised policy Division A (Gamma)

Division B (Gamma-plus)

--------------- Rupees ----------------

Contribution margin W.1 18,600,000 23,292,500

Fixed cost (7,000,000) (6,000,000)

Profit before Division managers' bonus 11,600,000 17,292,500

Bonus to division managers at 15% of profit after bonus (A) (1,513,043) (2,255,543)

Estimated profit - using revised policy 10,086,957 15,036,957

W.1: Determination of optimal option based on actual cost

Gamma (Division A) Gamma-plus (Division B)

Selling price

per kg

CM per kg (Sale-189

W.3)

Demand (kg)

Total CM (Rs.)

Selling price

per kg

CM per kg (Sale-627.25

W.3)

Demand (kg)

Total CM (Rs.)

300 111 150,000 16,650,000 960 332.75 70,000 23,292,500

375 186 100,000 18,600,000 1,080 452.75 50,000 22,637,500

450 261 50,000 13,050,000 1,200 572.75 30,000 17,182,500

W.2: Determination of the optimal option based on transfer price:

Gamma-plus (Division B)

Selling price per kg CM per kg

(Sale-671 – W.3) Demand (kg) Total CM (Rs.)

960 289 70,000 20,230,000

1,080 409 50,000 20,450,000

1,200 529 30,000 15,870,000

W.3 Actual variable costs/transfer price Division A Gamma

Division B Gamma Plus

----------Rupees----------

Inter-transfer cost from Division A (102+73)/2 - 87.5

Raw material (B) 637.50/2 102.00 318.75

Conversion costs - Variable and fixed 108.00 230.00

Conversion costs - Fixed (7/0.2), (6/0.25) (35.00) (24.00)

Conversion costs - Variable 73.00 206.00

Selling expenses - Variable 14.00 15.00

Total variable cost - Actual 189.00 627.25

Margin of profit to Division A on internal transfers (W-4) 87.5/2 43.75

Cost based on transfer price 671.00

(b) Profit using transfer price:

CM from external sale W.1 & W.2 18,600,000 20,450,000

CM from internal sale (50,000/2*87.5) 2,187,500 -

Total CM 20,787,500 20,450,000

Fixed costs (7,000,000) (6,000,000)

Profit before Division managers' bonus 13,787,500 14,450,000

Bonus to division managers at 15% of profit after bonus (B) (1,798,370) (1,884,783)

Estimated profit - using transfer price 11,989,130 12,565,217

Increase/(decrease) in bonus on revision of policy (A-B) (285,327) 370,760

W.4: CM per unit on internal transfer of Gamma to Division B: Maximum transfer price of Gamma (102+108)*1.25 262.50 Variable cost 102+73 175.00 Division A CM from internal transfers 87.50

MANAGEMENT ACCOUNTING Suggested Answers

Final Examination - Winter 2012

Page 6 of 6

A.6 (a) Fixed overheads variances calculation under absorption and marginal costing:

Absorption costing Marginal costing 1 Fixed overheads are allocated to

production at a standard fixed rate by using budgeted production.

Fixed overheads are treated as period cost and charged to profit and loss account.

2 Difference of actual and budgeted production results in under/over recovery of fixed overheads and a volume variance.

As fixed overheads are not allocated to production, volume variance does not arise.

3 Two variances are worked out i.e. expenditure and volume variances.

Only one variance is worked out i.e. expenditure variance.

(b) Ancient Pharma Limited

Profit statement for the month of November 2012

Standard costing using absorption:

Working Units Rupees

Sales - Export J (5,400*20%)*7,000*1.15 1,080 8,694,000

- Local, corporate

(5,400*25%)*7,000*90% 1,350 8,505,000

- Local, others

(5,400*55%)*7,000 2,970 20,790,000

5,400 37,989,000

Standard cost of sales

5,400*(4,000+800+500) (28,620,000) Gross profit 9,369,000 Favorable/(Adverse) variances: Material purchase price A 50,000*(486-500) 700,000 Material usage G,B (6,050*8)-49,000)*500 (300,000) Labour rate C 60,000*(80-88) (480,000) Labour efficiency H,C [(6,060*10)-60,000]*80 48,000 Variable overhead expenditure C,D,E [60,000*(50-20)-(3,500,000-1,213,250)] (486,750) Variable overhead efficiency H,C (6060*10-60,000)*(50-20) 18,000 Fixed overhead expenditure D,E (57,500*20)-1,213,250 (63,250) Fixed overhead volume variance H,D (6,060*10-57,500)*20 62,000

(502,000)

Gross profit after variances 8,867,000

Workings:

A Actual purchased quantity 24,300,000/ 486 kg 50,000 B Actual material usage 4,000+50,000-5,000 kg 49,000 C Actual labour hours used 5,280,000/88 Hours 60,000 D Standard fixed overhead rate (1,150,000/57,500) Rs./hr 20 E Actual fixed overheads (1,150,000*25%*1.10)+(1,150,000*75%*1.04) 1,213,250 F Finished goods production – Qty (100+6,300)-(150+250) Units 6,000

Equivalent units: G - Material 6,000-100+150 Units 6,050

H - Conversion costs 6,000-(100*60%) + (150*80%) Units 6,060 J Sales quantity 6,000+200-800 Units 5,400

(THE END)

MANAGEMENT ACCOUNTING Suggested Answers

Final Examination –Summer 2012

Page 1 of 6

A.1 (a) Himalaya Chemicals Limited

Introduction of the new product WYE

Selling price per unit 190.00 200.00 210.00

Direct material cost per unit (30.00)

Variable operating cost for dep’t. A and B W.1 (121.75)

(151.75) (151.75) (151.75)

Contribution margin per unit (A) Rs. 38.25 48.25 58.25

Demand in units (B) 18,000 15,000 12,000

Production at lower of demand or capacity of 16,800 units (12,600/0.75) (C) 16,800 15,000 12,000

Total contribution margin (A×C) Rs. 642,600 723,750 699,000

Renting out of spare operating facilities Dep't. A Dep't. B

Renting of spare operating hours (42,000×30%)

12,600.00 12,600.00

Rental income per hour

140.00 100.00

Variable operating cost per hour W.1 (111.00) (77.00)

Contribution margin per hour

29.00 23.00

Total contribution margin (12,600× (29+23))

Rs. 655,200

HCL should produce 15,000 units of WYE having selling price of Rs. 200 per units which gives the highest CM to the company.

W.1 Variable operating costs - WYE Dep't. A Dep't. B

Total overheads per hour 145.00 105.00

Fixed overheads per hour {1,356,600/ (42,000×95%)}, {1,117,200/ (42,000×95%)} (34.00) (28.00)

Variable operating cost per hour 111.00 77.00

Department-wise cost per unit (111*0.75), (77*0.5) 83.25 38.50

Total variable cost per unit

Rs. 121.75

(b) Other matters to be considered in deciding between the two options:

(i) Risk

Introduction of a new product WYE involves a risk that it may not be able to capture the market at the projected selling price.

(ii) Interference and difficulties with the tenant

In case of renting out of the facilities, the management may have to face day to day disputes and interferences.

(iii) Timely discontinuation of the contract

HCL may not be able to discontinue the arrangement in time when the facilities are needed for its own use.

MANAGEMENT ACCOUNTING Suggested Answers

Final Examination –Summer 2012

Page 2 of 6

A.2 Quality Appliances Limited

Material (kg) Labour hours Machine hours

Kg / Hours per unit HX (2,000/400) 5.00 (960/200) 4.80 (1,000/500) 2.00

HY (2,000/400) 5.00 (650/200) 3.25 (1,500/500) 3.00

Resources available 2,700 2,000 1,340

Objective function:

Maximise Contribution = *1,540HX+**

1,050HY subject to the constraints as mentioned below. *

HX contribution margin: 6,000-2,000-960-1,000-(625×80%)=1,540 **

HY contribution margin: 5,500-2,000-650-1,500-(375×80%)=1,050

Constraints:

Material 5HX+5HY ≤ 2,700 Eq. 1

Labour hours 4.8HX+3.25HY ≤ 2,000 Eq. 2

Machine hours 2HX+3HY ≤ 1,340 Eq. 3

HX, HY ≥ 100 Eq. 4

Plotting of constraints on the graph:

Number of units at feasible points A, B and C of the graph are worked out as below:

Point A

Putting the value of 100 for HX in equation 3, we get (2×100) + 3HY ≤ 1,340

∴HY = 380

Point B

Multiplying equation 3 by 2.4, we get 4.8HX+7.2HY=3,216 Eq. 5

Subtracting equation 2 from Equation 5 we get 3.95HY=1,216

∴HY = 308

Putting the value of HY in equation 5, we get 4.8HX+2,218=3,216

∴HX = 208

MANAGEMENT ACCOUNTING Suggested Answers

Final Examination –Summer 2012

Page 3 of 6

Point C

Putting the value of 100 for HY in equation 2, we get 4.8HX+(3.25×100) ≤ 2,000

∴HX =349

CM at feasible points A, B and C HX (Units) HY (Units)

CM (at Rs. 1,540 and Rs.

1,050 per unit)

A Including minimum required 100 units 100 380 553,000

B Including minimum required 100 units 208 308 643,720

C Including minimum required 100 units 349 100 642,460

QAL should produce 208 units of HX and 308 units of HY to maximise the contribution.

A.3 Spicy Foods Limited

BX BY BZ TOTAL

Units sold (in millions) A

0.400 0.600 0.300 1.300

Labour hours per unit B

1.50 1.75 2.00

Labour hours (in millions) C A×B 0.600 1.050 0.600 2.250

Required CM from sale of Jumbo pack to earn a profit of Rs 5 million

Rupees in million

Present sales D

140.000 180.00 126.00 446,000

Variable cost

Total cost of sales E

105.000 135.000 120.000 360.000

Fixed cost of sales

135/2.25×C (36.000) (63.000) (36.000) (135.000)

Variable cost of sales F

69.000 72.000 84.000 225.000

VC of sales reduced by 2%

F×98% 67.620 70.560 82.320 220.500

Variable operating costs G A×45, 49, 26 18.000 29.400 7.800 55.200

Revised total variable costs H

85.620 99.960 90.120 275.700

CM from present sales J D-H 170.300

CM from reduced 80% sales

(170.300/80%) 136.240

Fixed COS and operating costs 135+(30+49+13-55.2) (171.800)

Additional fixed cost (3+4) (7.000)

Required profit for the year (5.000)

(47.560)

Number of Jumbo packs to earn a profit of Rs. 5.0 million:

Sales price per Jumbo pack (350+300+420)×90% 963.00

VC of sales per Jumbo pack H/A 214.05 166.60 300.40 (681.05)

CM per unit per Jumbo pack Rs. 281.95

Number of Jumbo packs to be sold ( 47,560,000/281.95) Units 168,682

MANAGEMENT ACCOUNTING Suggested Answers

Final Examination –Summer 2012

Page 4 of 6

A.4 Sky Limited (a)

GAMMA (New facility)

ALPHA (Existing facility)

Option 1 Option 2 Option 3

Selling price per unit A 970.00 650.00 700.00 750.00

Variable cost per unit B W.1 (651.25) (485.00) (485.00) (485.00)

CM per unit C A-B 318.75 165.00 215.00 265.00

Hours per unit (existing facility) D 3.50* 2.00 2.00 2.00

CM per hour (Rs.) E C/D 91.07 82.50 107.50 132.50

Demand in units F 100,000 200,000 160,000 120,000

Production capacity:

Hours G 144,000 440,000 440,000 440,000

Units H (G/2.4) 60,000

G/2.0 220,000

G/2.0 220,000

(G/2.0 220,000

Demand in units J 100,000 200,000 160,000 120,000

Product having higher CM per hour will be produced first and restricted to lower of capacity / demand - units K

Note 1 Beta

60,000

Alpha 160,000

Alpha 120,000

Remaining capacity will be used for the products having lower CM per hour - units L

Alpha (G-(K*3.5)/2

115,000

Beta (G-(K*2)/3.5

34,286

Beta (G-(K*2)/3.5

57,143

Contribution margin; Alpha M

L*265 (Note.2)

30,475,000 K*C

34,400,000 K*C

31,800,000

Contribution margin; Gamma (at Rs. 318.75 per unit) N 19,125,000 10,929,663 18,214,331

Total contribution margin

49,600,000 45,329,663 50,014,331

*Hours for Beta only (conversion from Beta to Gamma would take place in the new facility) Conclusion

: SL should use option # 3 for producing 120,000 units of Alpha and 57,143 units of Beta and Gamma to maximise the profit.

Note 1 In case of option # 1 production of Beta / Gamma would be restricted to 60,000 units which is

the maximum capacity of the proposed new facility for producing Gamma.

Note 2 Alpha would be produced by utilising the capacity remained after producing Beta. As a result, production of Alpha will be restricted to 115,000 units and the sale price of Rs. 750 will be applicable as per option # 3 resulting in CM of Rs. 265 per unit.

W-1

Per unit variable cost of Alpha (590-(23,100,000/440,000×2) 485.00 Per unit variable cost of Gamma: Var. cost of Beta in the existing facility (735-23,100,000/440,000×3.5) 551.25 Var. cost of Gamma in the new facility (300-(12,000,000/144,000×2.4) 100.00 Var. cost of producing Gamma 651.25

(b) Transfer price of Beta

Before establishment of the proposed facility for Gamma, maximum contribution margin can be obtained from Alpha

Option # 2 160,000×215 34,400,000

After establishment of the proposed facility for Gamma, maximum contribution margin can be obtained from Alpha

Option # 3 120,000×265 31,800,000

Loss of CM due to decrease in production of Alpha 2,600,000

Loss of CM per unit of Beta 2,600,000 / 57,143 45.50

Variable cost per unit of beta in the existing facility W -1 551.25

Minimum transfer price per unit for Beta 596.75

MANAGEMENT ACCOUNTING Suggested Answers

Final Examination –Summer 2012

Page 5 of 6

A.5 Super Autos

LV MV

----- Rupees -----

Cost per unit under Activity Based Costing: Direct raw material 850,000/10,000; 900,000/12,000 85.00 75.00

Direct wages (3,000,000/30,000) ×1,000/10,000 10.00

(3,000,000/30,000) ×1,500/12,000

12.50

Overheads 798,750/10,000; 715,883/12,000 79.88 59.66

174.88 147.16

Cost per unit under the single factory overhead rate based on direct labour hours:

Direct raw material

85.00 75.00 Direct wages

10.00 12.50

Overheads *(8,600,000/30,000) ×1,000/10,000 28.67

*(8,600,000/30,000) ×1,500/12,000

35.83

123.67 123.33

*(2,500,000+1,500,000+2,000,000+1,000,000+1,600,000) = 8,600,000

W.1 Overheads allocation to LV and MV based on ABC method: LV MV

Production; 97×A 485,000 349200

Procurement 22,750×B 113,750 136,500

Finished goods stores and dispatch 16,458.33×8, 10 i.e. No. of sales orders 131,667 164,583

Quality control 1,366.67×C 68,333 65,600

798,750 715,883

LV MV

W.2 Machine hours A (25×200)= 5,000 (24×150) = 3,600

No. of purchase orders B (850,000/170,000)= 5 (900,000/150,000)= 6

No. of quality inspections C (10,000/400×2)= 50 (12,000/500×2)= 48

W.3 Computation of cost driver rate: Production Procurement F. goods stores & dispatch

Quality control

Machines operating expenses 2,500,000 - - - Maintenance expenses;

1,500,000×70%, 5%,15%, 10%

1,050,000

75,000

225,000

150,000 Technical staff expenses:

- Maintenance; (2,000,000×50%)×70%, 5%,15%, 10%

700,000

50,000

150,000

100,000

- Others; 2,000,000×30%, 12%, 0%, 8% 600,000 240,000 - 160,000 Procurement - 1,000,000 - - Finished goods stores & dispatch - - 1,600,000 - Overheads by department 4,850,000 1,365,000 1,975,000 410,000

Cost drivers

50,000 machine

hours

60 Purchase orders

120 Sales orders

300 (150×2) Quality

inspections Cost per driver 97 22,750 16,458.33 1,366.67

MANAGEMENT ACCOUNTING Suggested Answers

Final Examination –Summer 2012

Page 6 of 6

A. 6 Zen Trading Limited (a)

Present cost of recoveries:

Interest cost

Delay in collection Amounts collected Interest at 16%

Rs. in million

1 Month 26%×100×99% 25.740 0.343

2 Months 34%×100 34.000 0.907

3 Months 30%×100 30.000 1.200

5 Months 5%×100 5.000 0.333

11 Months 4%×100 ×80% 3.200 0.469 3.252

Discount cost for payment in one month 26×1% 0.260

Credit Control Department cost 1.2/12 0.100

Recovery cost of Credit Control Department 100×5%×10% 0.500

Immaterial balances written off 100×1% 1.000

Retention fee for the legal firm

0.025

Recovery fee of the legal firm 100×4%×80%×20% 0.640

Balances written off after legal proceedings 100×4%×20% 0.800

6.572

Cost of recoveries under factoring:

Factoring service charges at 5% 100×5% 5.000

Interest charges for recovery in 15 days 100×95%×16%×0.5/12 0.633

5.633

Savings on accepting factoring proposal 0.944

Conclusion: ZTL should accept the proposal of the factoring company as it will result in a saving of Rs. 944,000.

(b) Difficulties ZTL may have to encounter after accepting factoring arrangement. (i) There may be disagreements between the factor and the company, over credit evaluation of

the customer. (ii) A customer may feel uncomfortable in dealing with a factor. (iii) The factoring of trade debts may be considered as a symptom of weakness of the

management.

The above difficulties can be resolved by:

(i) Incorporating appropriate clauses in the agreement with the factor to avoid disagreements over credit evaluation of the customers.

(ii) Keeping continuous personal contacts with customers and resolving their problems promptly.

(iii) Effective public relationing with all the stakeholders.

(THE END)

MANAGEMENT ACCOUNTING Suggested Answers  

Final Examination –Winter 2011 

 

Page 1 of 6

A.1 Relevant costs for the tender Working

s Rs. in ‘000 Existing capacity's CM lost 1 6,913 Material A (8,000 × Rs. 820) 6,560 Material B (500 × Rs. 600) 300 Material B - loss on sale of redundant stock 2 30 Labour costs for project 3 3495 Factory overhead 4 2,752 Cost of machine 5 1,140

21,190 Profit margin on cost (20%) 4,238 Bidding price 25,428

W-1: Contribution margin lost

Capacity required for order 40% Present Idle capacity 25% Existing utilized capacity to be used for new project 15% Present quarterly contribution margin [Rs. 350,000 (Sales) - Rs. 234,500 (material) - Rs. 28,650 (labour inclusive of idle labour) - Rs. 23,000 (variable FOH) - Rs. 12,000(variable selling expenses)] 51,850

Contribution forgone for 60 days (15%/75% × Rs. 51,850 × 2/3) 6,913

W-2: Loss on sale of redundant Material B stock

Kgs Available stock 6,000 Usage during first 30 days (2,000) Usage during next 60 days (6,000 × 2/3×60/75) (3,200) Usage for new project (500) Redundant stock 300 Loss on sale of redundant stock at current sale price [(Rs. 700-600)×300) 30

W-3: Labour cost for the project

Skilled Unskilled Total hours required for project 6,000 15,000 Idle hours available Idle hours due to curtailment of activity from 75 to 60% 22,500 × 15/75 × 2 9,000 36,000 × 15/75 × 2 14,400 Present idle hours 22,500 × 10/90 × 2 5,000 36,000 × 4/96 × 2 3,000 14,000 17,400 Payment for the tender activity Rs. in ‘000 6,000 × 200 1,200 3,000 × 200 × 80% 480 14,400 × 125 1,800 600 × 125 × 20% 15

1,680 1,815 Total labour cost 3,495

MANAGEMENT ACCOUNTING Suggested Answers  

Final Examination –Winter 2011 

 

Page 2 of 6

W-4: Variable overheads of the project

Rs. in ‘000 Variable factory overhead rate 23,000/(58,500×3) 0.13105 No. of hours required for the new project 21,000 Variable overhead for two months 2,752

W-5: Cost of machine relevant for the project Purchased cost of machine 4,500

Less: Resale price or savings in labour costs whichever is higher:

Resale price 3,000

Savings in labour costs Machine life in # of months 36 Less: Project life 2 Life after the project 34 Labour costs for one months(Rs. 200×22,500+ Rs. 125× 36,000) 9,000 Savings in 34 months (Rs. 9,000 × 5% × 34) 15300 Less : 80% payable for idle time (12240) Add: Sale of old machine 300

3,360 3,360 1,140

A.2 (a) Sales volume variance

Products Total

XA YA ZA Budgeted sales (units) 60 28 20 Actual sales (units) 80 24 30 Volume variance in units (20) F 4 A (10) F × standard margin per unit ----- Rupees -----

XA : 200,000 - 39,500 - 80,000 - 100,000 (19,500)

YA : 300,000- 54,000 -100,000- 125,000

21,000

ZA : 475,000 - 78,000 -150,000- 187,500

59,500

Volume variance 390,000 A 84,000 A (595,000) F (121,000) F

(b) Sales price - planning variance Actual sales of XA at budgeted price (Rs. 200,000 × 80) 16,000,000 Revision in Budget Promotional Sales (180,000 × 35) (6,300,000) Other than Promotional Sales (200,000 × 45) (9,000,000) (700,000) A Sales price - operational variance Actual sales of XA [Rs.37,425,000 - (Rs.300,000 × 24) - (Rs. 475,000 × 30)] 15,975,000 Actual sales of XA at revised budgeted price (6,300 + 9,000 as above) (15,300,000) 675,000 F (c) Labour efficiency - planning variance Standard time XA: 80 × (80,000 ÷ 100) 64,000 YA: 24 × (100,000 ÷ 100 ) 24,000 ZA: 30 × (150,000 ÷ 100) 45,000 133,000 Revision in standard (8% improved performance) [133.000 × 92%] 122,360 Variance in hours (saved) 10,640 × standard rate per hour (Rs. 100 × 1.05) 105 1,117,200 F

MANAGEMENT ACCOUNTING Suggested Answers  

Final Examination –Winter 2011 

 

Page 3 of 6

Labour efficiency - operational variance Standard time (revised) 122,360 Actual time (120,000) Variance in hours 2,360 × standard rate per hour 105 247,800 F

A.3 Jan-12 Feb-12 Mar-12 Apr-12 May-

12 Total ----- Rupees in '000 -----

Receipts Mobilization advance 35,000 35,000 Running bills (RB) 42,000 63,000 35,000 - 140,000 Less: Mobilization advance (in the ratio of RB) (10,500) (15,750) (8,750) - (35,000) Retention money (5% of RB) (2,100) (3,150) (1,750) - (7,000)

29,400 44,100 24,500 98,000 Sales tax @16% of billing amount less mob adv 5,600 5,040 7,560 4,200 22,400 Withholding tax @ 6% (2,436) (2,066) (3,100) (1,722) (420) (9,744) Release of retention money - - - - 7,000 7,000 Net receipts 38,164 32,374 48,560 26,978 6,580 152,580 Payment of sales tax (5,097) (4,200) (9,297) Payment to supplier of equipments (95,000) (95,000) Installation charges (11,131) (25,972) - (37,103)

27,033 32,374 (77,509) 22,778 6,580 11,256

WORKINGS:

Input/output adjustment of sales tax Jan-12 Feb-12 Mar-12 Apr-12 Output tax as worked out above 5,600 5,040 7,560 4,200 Less: Input adjustment (16/116×95,000) (13,103) (7,503) (2,463) Paid / (excess) carried forward (7,503) (2,463) 5,097 4,200

Cost of installation and related works Contract price 140,000 Less: Profit margin @15% 21,000 Project cost 119,000 Less: Cost of equipments (100 / 116 × 95,000) 81,897 Cost of installation and related works 37,103

A.4 STATEMENT OF SAVINGS AND ADDITIONAL COSTS Workings Rs. in ‘000

Materials 1 5,329 Labour 1 56,481 Warranty 2 728 Factory overheads - ordering and holding costs 3 13,331 Savings in factory overheads (other than ordering & holding costs & depreciation) 4 5,264

Increase in factory overheads due to increase in depreciation (9,000-2,700) (6,300) 74,833

Conclusion: Implementation of the consultants' suggestions would increase the gross margin by more than 17% (74,833 ÷ 434,000 = 17.2%) and therefore consultants' suggestions should be accepted. However, their suggestion as regards JIT system is not feasible as against the savings of Rs.13.3 million on inventory ordering and holding costs, discount on bulk purchases amounting to Rs.14.5 million would be lost.

MANAGEMENT ACCOUNTING Suggested Answers  

Final Examination –Winter 2011 

 

Page 4 of 6

W-1 : Savings in cost of labour and material on account of purchase of new machine Material Labour

Cost of labour and material if new machine is NOT purchased Rs. in ‘000 Current years cost (A) Note: 26,000 units = 3540 + 24960 – 2500 834,400 138,600 Cost to produce 2000 additional units in next year at current rates (2,000 units / 26,000 × A) 64,185 10,662 Cost to produce 28,000 units at current rates 898,585 149,262 Impact of 8% increment in material and 10% increase in labour costs 71,887 14,926 Cost of labour and material if new machine is NOT purchased 970,472 164,188

Cost of labour and material if new machine is purchased Cost of material before wastage (970,472 ×0.96) 931,653 Wastage under new machine (931,653 ×2 /98) 19,013 Cost of material after discount 950,666 Loss of discount if bulk purchases discontinued (950,666×1.5/98.5) 14,477 Skilled labour (164,188 × 8 /25) 52,540 Unskilled labour (164,188 /25 ×12× 70%) 55,167

Cost of labour and material if new machine is purchased 965,143 107,707 Net savings in labour and material costs 5,329 56,481 W-2 : Savings in cost of warranty Rs.in‘000 Costs under present conditions (Rs. 4,000 × 28000 × 1%) 1,120 Less: Costs if new machine is purchased (Rs. 3,500 × 28000 × 0.4%) 392 Net savings 728

W-3: Savings in ordering/holding costs

Existing Proposed Size of order (B) 14,000 [14,000 × (1-85.71%)] 2,000 No. of orders (28,000 ÷ B) (C) 2 14 Avg. inventory (B/2+ 4,000) (D) 11,000 (B/2) 1,000

Rs.in‘000 Rs.in‘000 Ordering costs (Rs. 45,000 × C) 90 630

Holding costs (970,472 × D / 28000 × 4%) 15,250 (965,144 × D/28,000 x 4%) 1,379

Total ordering and holding costs 15,340 2,009

Savings in ordering and holding costs (Rs. 15,340 - Rs. 2,009) 13,331

W-4: Factory overheads Existing

As per annual accounts 193,502 Less: Depreciation (Rs. 54 - Rs. 5.4) /18 (2,700) Ordering and holding costs as above (15,340)

175,462 Savings in factory overheads {175,462x(10%-7%} (other than depreciation and inventory ordering and hold costs 5,264

MANAGEMENT ACCOUNTING Suggested Answers  

Final Examination –Winter 2011 

 

Page 5 of 6

A.5 Process Account - Department 2 Units Rs. Units Rs.

Opening work in process 2,000 128,750 Normal loss account (W-1) 2,500 37,500 Received from Dept 1 53,000 2,057,500 Transferred to Dept 3(W-1) 48,000 3,598,750 Direct Material 988,000 Closing work in process (W-1) 5,000 307,500 Direct wages 488,000 Production overheads 244,000

Abnormal gain (500 × 75) 500 37,500

55,500 3,943,750 55,500 3,943,750

W-1: Costs computation Rs.

Cost of units transferred to Department 3 Opening WIP: Balance as at November 1, 2011 (2,000 units) 128,750

Material (400 [W-2] x Rs. 20 [W-3]) 8,000

Labour & Prod OH (800 [W-2] x Rs. 15 [W-3]) 12,000

148,750 Introduced and completed within the month (46,000 x Rs. 75) 3,450,000

3,598,750

Cost of closing work in process Transferred from Department 1 (5000 x Rs. 40 ) 200,000

Material (3,500 [W-2]xRs.20 [W-3]) 70,000

Labour & Prod OH (2,500 [W-2]xRs.15 [W-3]) 37,500

307,500

W-2 : Equivalent Production Unit (EPU)

Total Transferred from dept 1

Material introduced in dept 2

Labour & Prod. OH

------------ Units ------------ Opening WIP 2,000

Transfer in 53,000 55,000

Accounted for Opening WIP Completed 2,000 - 400 800 Started and completed 46,000 46,000 46,000 46,000 Closing WIP 5,000 5,000 3,500 2,500 Normal loss [(55,000 - 5,000)×0.05) 2,500 - - - Abnormal gain (2,500 - 2,000) (500) (500) (500) (500)

55,000 50,500 49,400 48,800

W-3 : Cost per unit for each element

Cost (Rs.) EPU (W-2)

Cost per unit (Rs.)

Transfers from dept 1 2,057,500 Less: Scrap value of normal loss (2,500 × Rs. 15) 37,500

2,020,000 50,500 40 Material 988,000 49,400 20 Wages 488,000 48,800 10 Overheads 244,000 48,800 5 Total costs per unit 75

MANAGEMENT ACCOUNTING Suggested Answers  

Final Examination –Winter 2011 

 

Page 6 of 6

A.6

Computation of total hours required for 1st and repeat orders

Rupees Sale price per unit 10,500 Less: Margin @ 20% 2,100 Cost 8,400 Less: Costs not affected by learning curve (W-1) 4,250 Costs dependent on learning curve A 4,150

Variable cost (labour & overheads) per hour–Dept B (Rs. 200×1.25)

B 250

Avg. labour hours per unit for 1st and repeat order C = A / B 16.60 Labour hours per unit for 1st order D 20 Learning curve factor E = C / D 0.83 Relevant cumulative total volume factor as per table F 1.80 Units for 1st order G 500 Total units for 1st and repeat order H (F × G) 900 Repeat order J (H-G) 400

Applied fixed overheads - to be ignored

Working 1 Costs not to be affected by learning curve Direct Material 3,350 Direct labour in Department A 720 Variable overheads (25% of labour cost in Department A) 180

4,250

(THE END)

MANAGEMENT ACCOUNTING Suggested Answers

Final Examinations –Summer 2011

A.1 (a) Budgeted sales 8,250,000 / 275 units 30,000 Sale price variance 275 × 10% ×

(30,000/3x1.2) Rs. 330,000 A

Actual sales (8,745,000 + 330,000) / 275

units 33,000

Sale volume variance (33,000 – 30,000) × 275 Rs. 825,000 F Material price variance – A (36.4 – 35) × 63,900 × 2/3 Rs. 59,640 A B (20.8 – 20) × 105,600 × 2/3 Rs. 56,320 A Total material price variance Rs. 115,960 A Material mix variance Actual mix at standard cost- Material A 63,900 × 35 Rs. 2,236,500 - Material B 105,600 × 20 Rs. 2,112,000 Rs. 4,348,500 Standard mix* (35 × 2 + 20 × 3) 169,500 × (130* / 5) 4,407,000 Mix variance Rs. 58,500 F Material yield variance: Standard output from actual input 169,500 / 5 units 33,900 Actual output units 33,000 Yield variance units 900 s Yield variance 900 × 130 Rs. 117,000 A Labour rate variance Standard rate per hour 2,700,000 / (30,000 × 1.5) Rs. 60 Actual average rate per hour (60 + 66 + 66) / 3 Rs. 64 Actual labour hours worked 3,041,920 / 64 hours 47,530 Labour rate variance 47,530 × (64-60) Rs. 190,120 A Labour efficiency variance: Standard labour hours required 33,000 × 1.5 49,500 Actual labour hours used 47,530 Efficiency variance hours 1,970 F Labour efficiency variance 1,970 × 60 Rs. 118,200 F

(b) (1) Sales price variance: The variance arose on account of discount allowed by the company in March 2011. The decision to allow this discount does not seem appropriate on account of the following: The company had already managed to surpass the budgeted sales quantity in January & February 2011. The raw material prices and labour rate have seen increased from February, 2011. After allowing discount, the sale in March made less contribution margin as compared to February. (2) Yield variance: Although there could have been more than one reason for the above variance, it may have been due to.

use of cheaper materials as indicated by favourable mix variance. Another important point is that the adverse yield variance may have been partly on account of poor labour performance. This aspect needs to be investigated.

(3) Adverse labour rate variance: It was on account of increase in wages which was a conscious decision of the management. It seems that in

the short run i.e. in February and March the extra cost could not be recouped by the increased efficiency. However, in the long run it may have far reaching positive effect.

MANAGEMENT ACCOUNTING Suggested Answers

Final Examinations –Summer 2011

Punjnad Juice Company statement for projected working capital: A.2 No. of Units Year 1 Year 2 Opening stock A 1,000 Production B 15,000 20,000 Sales C 14,000 18,000 Closing stock D 1,000 3,000 ---------------

-- Rupees -------------------

- Current Assets: Stock – Raw material (B × 80 × 3/12) 300,000 400,000 Finished goods W-1 168,000 469,714 Debtors rate of provision for doubtful debts (200 ×

C/12)X0.9925 231,583 297,750

699,583 1,167,464 Current liabilities: Creditors for supply of raw material W-2 250,000 283,333 Creditors for expenses W-3 65,833 72,667 315,833 356,000 Working capital required – Net 383,750 811,464 W-1 Finished goods Materials ( B x 80) 1,200,000 1,600,000 Labour & variable overheads (B × 40) 600,000 800,000 Fixed overheads (20 × 24,000) 480,000 480,000 Depreciation (10 x 24,000) 240,000 240,000 Cost of goods manufactured E 2,520,000 3,120,000 Finished goods {E/B×1,000} and {(E+168,000)/(B+A) ×D} 168,000 469,714 W-2 Creditors for raw material Material consumed(B × 80) 1,200,000 1,600,000 Closing stock (1,200,000 x 3/12) 300,000 400,000 Opening stock - -300,000 Purchases during the year F 1,500,000 1,700,000 Creditors outstanding (F × 2/12) 250,000 283,333 W-3 Creditors for expenses Variable overheads (10 × B/12) 12,500 16,667 Fixed overheads (20 × 24,000/12) 40,000 40,000 Fixed selling expenses (2 × 24,000/12) 4,000 4,000 Variable selling expenses (8 × C/12) 9,333 12,000 65,833 72,667

After option 2 A.3 W-1 Burger Fries Cold drink Ice cream Total Sale price per unit A 150 50 40 80 Sale ratio B 6 7 8 3 Weightage (A × B) C 900 350 320 240 1810 Item wise Sale (Rs. in million) D 90 35 32 24 181 Contribution % E 40% 45% 50% 60% Contribution Margin (Rs. in million) (D × E) F 36.00 15.75 16.00 14.40 82.15

Average contribution margin % (82.15/181×100) 45.39%

Cost per deal{A × (100–E)} 90.00 27.50 20.00 32.00 137.50

Cost per Deal 2 90.00 27.50 20.00 32.00 169.50

MANAGEMENT ACCOUNTING

Suggested Answers Final Examinations –Summer 2011

OPTION – 1 Rupees Sale of individual items 30% ×(1.25x181 million) 67,875,000 Sale of deals 70% ×(1.25x181 million) 158,375,000 Total sale @ 25% (1.25 × 181 million) 226,250,000 Contribution Sale CM % W-1 30,808,463 Individual items 67,875,000 45.39 W–1 32,802,630 Deal 1 (60% of 158,375,000) 95,025,000 34.52 W–1 24,997,910 Deal 2 (40% of 158,375,000) 63,350,000 39.46 88,609,003 Total contribution from option I 226,250,000 W – 1: Contribution margin on Deals Deal 1 Deal 2 Selling price 210.00 280.00 Cost of deals 137.50 169.50 Contribution margin 72.50 110.50 Contribution margin % 34.52 39.46 OPTION - 2 Total sale (1.35 × 181 million) 244,350,000 Home delivery sale 30,000,000 Outlet sales 214,350,000

Sale

price Sale ratio Weight-age

Item wise Sale Contri-bution

%(W.2) Contribution

Margin Outlets Home Delivery Total

Burger 120 6 720 106,582,873 17,197,452 123,780,325 25.00 30,945,081 Fries 40 7 280 41,448,895 6,687,898 48,136,793 31.25 15,042,748 Cold drink 32 8 256 37,896,133 6,114,650 44,010,783 37.50 16,504,044 Ice cream 64 3 192 28,422,099 - 28,422,099 50.00 14,211,050 1448 214,350,000 30,000,000 244,350,000 76,702,923

Additional cost on home delivery sets: Variable cost of home delivery 30,000,000/600*20 Fixed cost of home delivery Total contribution from option II

(1,000,000)

74,852,923 (850,000)

W-2 COMPUTATION OF REVISED CONTRIBUTION MARGIN PER UNIT

Sale price (Rs.)

Present cost (Rs.) Revised CM (Rs.) CM %

Burger 120 90 30.00 25.00 Fries 40 27.5 12.50 31.25 Cold drink 32 20 12.00 37.50 Ice cream 64 32 32.00 50.00

Conclusion: Option 1 is more profitable A.4 MW Oct-Mar Apr-Jly Aug-Sep Total W-1 Factory consumption capacity A 4.00 Non-production use B 0.25 0.25 0.25 0.25 Factory consumption @100 cap. C 3.75 Machine / capacity usage D 1/3×80% 2/3x90% 3/3×100% Factory consumption (C×D) –

MW E 1 2.25 3.75

Total consumption F 1.25 2.5 4 Available for sale (5.0 Mw – F) G 3.75 2.5 1 Number of DAYS H 180 120 60 Chargeable units to utility 16,200,000 7,200,000 1440,000 24,840,000

MANAGEMENT ACCOUNTING Suggested Answers

Final Examinations –Summer 2011

Computation of total and per megawatt costs: Rupees Operational (12*1,500,000) 18,000,000 Labour Cost (12x250,000) 3,000,000 Other related costs (12x500,000) 6,000,000 Depreciation (Rs 100 - 4 million) / 6 years 16,000,000 Fuel Cost (12x24,000,000) 288,000,000 Total generation cost 331,000,000 Financing cost of interconnecting structure (15M*16%) 2,400,000 Profit required 60,000,000 393,400,000 Less: Amount presently being paid 180,000,000 Amount to be charged from utility company 213,400,000 Chargeable units to the utility company to be charged to the utility company 8.59

Landed cost price of good unit A.5 W-1 FML –

Pak LMN China PQR

Singapore ----------- -- Rupees- ------------- Purchase price per units 287.50 265.00 280.00 Freight-in per unit 2.00 9.00 5.00 C & F value space 289.50 274.00 285.00 Import duty at 10%{(126.50/1.15)-100} - 27.40 28.50 289.50 301.40 313.50 Cost per good unit (289.50 / 0.93) (301.40/0.99)

(313.50/0.99) 311.29 304.44 316.67 Recovery through sale of defective rang material @Rs.4 (40/100)(40/100)

(40x0.0.07) / (40x0.01) per unit (2.80) (0.40) (0.40) 308.49 304.04 316.27 Purchasing priority 2 1 3

W-2 ANNUAL REQUIREMENT OF PRIMARY RAW MATERIAL AND QUANTITIES OUGHT TO BE PURCHASED FROM EACH SUPPLIER Capacity of Department B net of normal losses

Units 4,000,000

Output loss -(5/95*4,000,000) 210,526 required in department B 4,210,526

Material required to be put into Department-A to get above output (4,210,526/90%) 4,678,362

Totals Defective Good C&F per units

Cost

To be purchased from LMN

Rupees

2,000,000 20,000 1,980,000 301.40 602,800,000

To be purchased from FML 1,600,000 112,000 1,488,000 289.50 463,200,000

To be purchased from PQR (Balancing) 1,222,588 12,226 1,210,362 313.50 383,281,338

144,226 4,678,362 1,449,281,338 Proceeds from sale of scrap units (144226*40) (5,769,040) Cost material 1,443,512,298

Statement of department wise cost Quantities

Received from preceding dept(4000,000/0.95)

: -------------------No. of units--------------------- 4,210,526

Put into department (4,210,526/0.9) 4,678,362

Process losses (467,836) (210,526)

4210,526 4,000,000

Cost uncured: Depart-B Depart-B

Rupees

Raw material/cost from provably dept W-2 1,443,512,298 1,818,635,094

Wages (4,210,526 × 18/60×200);(4,210,526 ×12/60×250) 252,631,560 210,526,300

Variable over heads (60% and 75% of wages) 151,578,936 157,894,725 Fixed over heads (10M ×1,263,158/2,105,263)(10M×842,105/2,105,263) 6,000,000 4,000,000

Sales of Scrap (75×467,836)(125×210,526) (35,087,700) (26,315,750) Total cost of goods manufactured 1,818,635,094 2,164,740,369 Cost per unit 431.93 541.19

A-6 LINEAR PROGRAMMING

Variables

Let x = number of tables

Let y = number of chairs

Constraints

Machine hours x + 0.5y ,<=600 Eq. 1 Labour hours 1.5x + 2y <=1,890 Eq. 2 2x <= y Eq. 3 x>=100 Eq. 4

Machine Labour Available hours 715 2,250 Less: Hours required for confirmed order

Tables (40) (60) Chairs (75) (300) (115) (360) 600 1,890 Contribution margin per unit Table Chair

Sale price 2,300 900 Cost of sales

Material 1,000 300 Machine cost 450 225 Labour 90 120 Other manufacturing cost 200 50 1,740 695

560 205

Objective function is to maximize 560x + 205y

x + 0.5y =600-------------------- Eq. 1 if y = 0, x = 600; if x = 0, y = 1,200

1.5x + 2y = 1,890 ----------------Eq. 2 if y = 0, x = 1,260; if x = 0, y = 945

Optimal solution is at points P,Q and R Point PPutting the value of x = 100 i.e. Equation 2, in Equation 4 we get:

can be ascertained by solving equation 2 and 4

150 + 2y = 1,890 y = (1,890 - 150) /2 = 870 Point QMultiplying Eq. 1 by 4, we get 4x +2y = 2,400 ------- Eq. 5

can be ascertained by solving equation 1 and 2

Subtracting Equation 5 from Equation 2, we get: 2.5x = 510 x = 204 Putting the value of x in Equation 1 we get: 204 + 0.5y = 600 ∴y = 792 Point Rx + 0.5y = 600

can be ascertained by solving equation 1 and 3

2x = y OR x = 0.5y 0.5y +0.5y = 600 y = 600 Putting the value of y in equation 1 we get: x + 300 = 600 ∴x = 300 Putting the above values in objective function we get: At point P: x = 100 × 560 = 56,000; y = 870 × 205 = 178,350; Total = 234,350 At point Q: x = 204 × 560 = 114,240 y = 792 × 205 = 162,360; Total = 276,600 At point R: x = 300 × 560 = 168,000; y = 600 × 205 = 123,000; Total = 291,000

Contribution is maximized at point R, profit = Rs. 291,000

(THE END)

MANAGEMENT ACCOUNTING Suggested Answers

Final Examinations –Winter 2010

Page 1 of 6

A.1 (a) (i) Budgeted cost and sales price per set Rupees C & F value 9,500

Import related costs and duties 900 Variable cost of local value addition 3,500 Variable cost per set 13,900 Fixed production overheads (Rs. 12,000,000/5,000 sets) 2,400 Budgeted cost of production per set 16,300 Add: Gross profit (Rs. 16,300 × 25%) 4,075 Budgeted sales price per set to distributor 20,375 Budgeted gross profit (Rs 4,075 × 5,000 sets) 20,375,000 Less: Admin & selling expenses Variable (Rs. 900 × 5,000 sets) (4,500,000) Fixed (9,000,000) Budgeted annual profit 6,875,000 (ii) Computation of budgeted consumer price of each set Budgeted sales price of the company 20,375.00 Add: distributor margin (Rs. 20,375 × 10/90) 2,263.88 Budgeted sales price of the distributor 22,638.88 Add: wholesaler margin (Rs. 22,638.88 × 4/96) 943.29 Budgeted sales price of wholesaler 23,582.17 Add: retailer’s markup (Rs. 23,582.17 × 10%) 2,358.21 Budgeted retail price 25,940.39 Revised retail price (Rs. 25,940.39 × 95%) 24,643.37 Revised profit forecast after considering consultants’ recommendation: Rupees Sales (6,500 sets × Rs. 24,643.37) 160,181,905 Less: Cost of goods sold for 6,500 units Electronic Kits @ Rs 9,500 61,750,000 Cost of import and duty @ Rs 900 5,850,000 Local value addition @ Rs 3,500 22,750,000 Fixed overhead cost 12,000,000 (102,350,000) Gross Profit 57,831,905 Less: Selling & Admin expenses Variable (6,500 sets × Rs 900) 5,850,000 Fixed 9,000,000 Cost of advertisement campaign 5,000,000 Cost of after-sale service (6,500 × Rs. 450) 2,925,000 Retailers commission (Rs. 160,181,905 × 15%) 24,027,285 (46,802,285) Profit by implementing the proposal of consultant 11,029,620

Based on above results, management should accept the recommendation of the consultant.

MANAGEMENT ACCOUNTING Suggested Answers

Final Examinations –Winter 2010

Page 2 of 6

(b) In the light of the changes recommended by the consultant, the company will have to consider whether it has the necessary infrastructure to:

(i) deal with a far larger number of retailers as against the present few distributors. (ii) produce and sell extra 30% t.v. sets.

(iii) attend to after sale activities on its own. The question is silent as to who presently

attends to this activity. (iv) conduct effective advertisement campaign.

Fixed expenses related to manufacturing as well as selling and admin are likely to increase but no such increase has been anticipated.

A.2 Formula of learning curve baxy = Where a is the labour cost for the 1st

lot

x is the cumulative number of lots b equals log(0.9)/log(2) = −0.152 Average cost Total cost For 1,000 units (10 lots) -0.15210 20,000 y ×= 14,093.86 140,939 For 46,000 units (460 lots) -0.152460 20,000 y ×= 7,875.73 3,622,836 For 50,000 units (500 lots) -0.152005 20,000 y ×= 7,776.54 3,888,270 For 49,900 units (499 lots) -0.152499 20,000 y ×= 7,778.91 3,881,676 Cost for the last 100 units (500th lot) 6,594

45,000 units

(450 lots) 70,000 units

(700 lots) Revenue 29,250,000 38,500,000 Material cost 13,500,000 21,000,000 Labour upto 460 lots 3,622,836 - Labour upto 5oo lots - 3,888,270 Labour for additional 210 lots - 1,384,740 Less: labour upto 10 lots (140,939) (140,939) 3,481,897 5,132,071 Variable overheads – 25% of labour cost 870,474 1,283,018 Fixed cost 1,300,000 1,300,000 19,152,371 28,715,089 Net profit 10,097 ,629 9,784,911

Conclusion: It will be more profitable to sell 45,000 units at a unit price of Rs. 650.

MANAGEMENT ACCOUNTING Suggested Answers

Final Examinations –Winter 2010

Page 3 of 6

A.3 (a) Carrying costs: Rupees Interest @ 8.0% on Rs. 2,400 192 Insurance @1% on Rs. 2,400 24 216 Costs associated with each order: Purchase department costs 3,000 Cost of delivery 6,000 9,000 Carrying costs per unit for special order: Interest @ 8.0% on Rs. 2,400 x 95% 182.40 Insurance @1% on Rs. 2,400 x 95% 22.80 205.20

Determining stock-out and carrying costs:

Stock level

Incremental stock

Stock out Probability Loss of

sales

Loss of margin

(1,000x)

Carrying costs

(216x) Total

900 100 0.45 45 45,000 200 0.20 40 40,000 300 0.05 15 15,000 100,000 - 100,000 1,000 100 100 0.20 20 20,000 200 0.05 10 10,000 - 30,000 21,600 51,600 1,100 200 100 0.05 5 5,000 43,200 48,200 1,200 300 - - 64,800 64,800 Therefore, the company should reorder when the stock reaches 1100 units. (b) Units Probability Expected monthly sales 900 0.30 270 1,000 0.45 450 1,100 0.20 220 1,200 0.05 60 1,000 Expected yearly sales 12,000 ( ) units000,1216/000,9000,122 EOQ =××=

Annual costs: EOQ Special offer

Total cost Rupees

Cost of purchase (2,400 x 12,000) 28,800,000 27,360,000 (2,400 x 95% x 12,000)

Ordering costs (9,000 x 12) 108,000 18,000 (9,000 x 2) Carrying costs 1,000 / 2 x 216 108,000 615,600 6,000 / 2 x 205.20 29,016,000 27,993,600 Conclusion: FL should accept the offer

MANAGEMENT ACCOUNTING Suggested Answers

Final Examinations –Winter 2010

Page 4 of 6

A.4 (a) Return on capital employed Division A Division B Divisional profit as per question 480,000 115,000 Capital employed: Assets 2,433,100 643,000 Less: Liabilities 638,000 234,600 Net assets 1,795,100 408,400 Return on capital employed 26.74% 28.16% Residual Income method Net profit 202,400 48,900 Add: Apportioned head office cost 243,000 45,600 445,400 94,500 Return on equity (A × 20%) 304,020 48,680 Residual Income 141,380 45,820 Equity(Assets-Liabilities) [2,433,100-913,000] [643,000-399,600] (A) 1,520,100 243,400 Division B has slightly higher return but on account of the far larger size of division A, this

small difference does not definitely indicate better performance from division B. Division A is contributing 80% of the company’s profit and seems more important for the company.

However, in the absence of information such as nature of business, nature and magnitude

of business risks and reasons for significant differences between debt equity ratios of the two divisions, a meaningful comparison is difficult. It would be advisable for the company to compare the returns with those of similar industries in the region.

(b) (i) Return on capital employed Rs. in ‘000 Net profit from new investment 6,000 Add: financial charges (90m × 50% × 14%) 6,300 Return on net investment 12,300 Net investment (80+10 −16 being depreciation) 74,000 Return on capital employed 16.62% Residual income Rs. in ‘000 Net profit on investments 6,000 Return on equity [29,000(W-1) × 20%] 5,800 Residual income 200 W–1: Increase in assets 74,000 Less: increase in liabilities (90 ÷ 2) 45,000 Increase in net assets 29,000 ROCE 25%

In view of low return i.e. 16.62% on new investment compared to existing return of 26.74%, the project will not be acceptable to the divisional manager.

However, the return of 16.62% pertains to the first year only. In future years, the net investment would reduce and hence return percentage would increase significantly. If the divisional manager takes a long term perspective, he may decide to accept the proposal.

MANAGEMENT ACCOUNTING Suggested Answers

Final Examinations –Winter 2010

Page 5 of 6

In view of positive residual income, the project will be beneficial for the company.

Although ROCE and residual income are accepted as proper short-term measures for large investment/disinvestment projects and for inter-division comparisons, the evaluation must also consider NPV and DCF methods which take account of time value of money as well as cover the entire life of the project.

(ii) Return on capital employed: Impact on return Contribution loss (9,000) Reduced depreciation 20,000 Operating profit 11,000 Loss on sale of asset (12,000) Net loss (1,000) Capital employed: Asset – no change* - Decrease in current liabilities (50% of 8.0 million) 4,000 Increase in capital employed 4,000

Residual income method: Net loss before financial year as computed above (1,000) Less: financial charges saved (14% of 4.0 million) 560 Net loss (440) Increase in required return from equity (8,000**x20%) (1,600) Residual income (2,040)

If the company follows a policy whereby gain/loss on sale of asset do not form part of the division’s performance, the divisional profit would increase by Rs. 11 million whereas the capital employed would increase by Rs. 4 million only and hence sale of asset would be favourable from the divisional manager’s point of view.

However, if gain or loss on sale of asset is treated a part of divisional performance then as a result of sale, the divisional profit would be reduced by Rs. 1.0 million whereas capital employed would increase by Rs. 4.0 million. Hence in this case the sale of asset would not be favoured by the divisional manager.

From the CEO’s point of view, the sale would not be acceptable in any case as it would

contribute to a net reduction in residual income of the company by Rs. 2.04 million. A.5 Since chemicals X & Y are produced from extracts P and Q which are produced in a joint

process and in a specified ratio, the production of each chemical is interdependent. Based on the market demand, the company has 2 options as follows:

Production Option1: Produce 600,000 litres of product X and 160,000 Litres of product Y

(Working 1) Production Option2: Produce 180,000 litres of product Y and 600,000 litres of product X and

dispose off any excess quantity of extract P in raw form D. (Working 2) Working 1 Litres Quantity of P required to produce 600,000 litres of chemical X (600,000 ×1/5) 120,000 Qty of Q produced in the joint process 1 (120,000 × 2/3) 80,000 Hence quantity of Y to be produced = (80,000 × 2) 160,000

MANAGEMENT ACCOUNTING Suggested Answers

Final Examinations –Winter 2010

Page 6 of 6

Working 2 Quantity of Q required to produce 180,000 litres of chemical Y (180,000 ×1/2) 90,000 Quantity of P produced in the joint process (90,000 × 3/2) 135,000 Quantity of P required to produce 600,000 litres of X being maximum demand for P 120,000 Surplus quantity of P to be sold without further processing 15,000

Option 1 Option 2 Litres Cost Litres Cost Raw Material Costs Quantity to be produced -P 120,000 135,000 Q 80,000 90,000 Total 200,000 225,000 Wastage (Total Qty / 0.2 × 0.8) 50,000 56,250 Inputs required in Process 1 (A) 250,000 281,250 Material A 2/3 166,667 4,166,667 187,500 4,687,500 Material B 1/3 83,333 3,333,333 93,750 3,750,000 Material C (Qty of Ext P × 4) 480,000 36,000,000 480,000 36,000,000 Material D (Qty of Ext Q × 1) 80,000 4,400,000 90,000 4,950,000 Total Direct Labour - Process 1 80×(A) 20,000,000 22,500,000 Direct Labour - Process 2 (50×(600+160)) 38,000,000 (50×(600+180)) 39,000,000 Variable overheads - Process 1 (57×250) 14,250,000 (57×281.25) 16,031,250 Variable overheads - Process 2 (32×(600+160)) 24,320,000 (32×(600+180)) 24,960,000 Total variable Costs 144,470,000 151,878,750 Option 1 Option 2 Profit & Loss Sale of X (600000×250) 150,000,000 150,000,000 Sale of Y (160000×450)/(180000×450) 72,000,000 81,000,000 Sale of P(15000×100) 1,500,000 Sales revenue 222,000,000 232,500,000

Variable cost of production

(144,470,000) (151,878,750

) Fixed costs (5,000,000) (5,000,000) Net profit 72,530,000 75,621,250 Option 2 produces higher profit.

(THE END)

MANAGEMENT ACCOUNTING

Suggested Answers Final Examinations – Summer 2010

Omair Jamal Page 1 of 7

A.1 Price Units Amount (Rs. ‘000s) Men Women Men Women Men Women Minimum 1,000 800 720,000 300,000 720,000 240,000 Maximum 4,000 2,500 120,000 50,000 480,000 125,000 Average 2,000 1,200 360,000 150,000 720,000 180,000 Total 1,200,000 500,000 1,920,000 545,000 Rs. 000s Sales revenue – gross (1,920,0000 + 545,000) 2,465,000 Less : Commission to distributors 20% ×30% of above 147,900 Cut size discount 40% × (5% of 70%) 34,510 182,410 Sales – net 2,282,590 Variable cost 100/220 of gross revenue 1,120,455 1,162,135 Less : Factory overheads 12 × 45m 540,000 Gross profit 622,135 Less : Admin overheads 12 ×15m 180,000 Cost of retail outlets 12 × 22 × 1.2m 316,800 496,800 Net profit 125,335 A.2 Objective function: Maximize Z = 150,000x + 100,000y Current constraints: 120x + 80y = 18200 Eq 1 if y = 0, x ≤ 151; if x = 0,y ≤ 227 80x + 50y= 12000 Eq 2 if y = 0, x = 150; if x = 0,y = 240 x>0 and y>0 600x + 400y= 91000 Eq 3 Eq 1×5 640x + 400y 96000 Eq 4 Eq 2×8 −40x= −5000 x= 125 80x +50y= 12000 Eq 2 10000 +50y= 12000 50y= 2000 y= 40 Revised constraints 120x + 80y = 18400 if y = 0, x ≤ 153; if x =0,y = 230 80x + 50y= 12000 Eq 2 if y = 0, x = 150; if x =0,y = 240 x>0 and y>0 600x + 400y= 92000 Eq 3 Eq 1×5 640x + 400y 96000 Eq4 Eq2 × 8 −40x= −4000 x= 100 80x +50y= 12000 Eq 2 8000+50y= 12000 50y= 4000 y= 80

MANAGEMENT ACCOUNTING

Suggested Answers Final Examinations – Summer 2010

Omair Jamal Page 2 of 7

Current Options: Production Contributions x y A 150 0 22,500,000 B 125 40 22,750,000 C 0 227 22,700,000 Required options: Production Contributions x y A 150 0 22,500,000 B 100 80 23,000,000 C 0 230 23,000,000 Shadow price for additional capacity: (23,000,000 - 22,750,000) = 250,000/200 = Rs.1,250 per hour A.3 Cash Management

Total sales Units Weight Sales Ratio Cash sales – 25% 6,000 1.0 6,000 Credit sales – 75% 18,000 1.1 19,800 24,000 25,800 Sales Revenue (Rs. in ‘000) 51,600 Cash Selling price per unit 2,000 Credit selling price per unit 2,200 Cash Requirement 2010 -11 Particulars Qtr. 1 Qtr. 2 --- Rs. in ‘000 --- Purchase of machinery (60,000) - Sale receipts - - Cash sales (2,000 × 6,000 / 4 × 94%) 2,820 2,820 Receipts from credit sales – as per working below 5,211 9,120 Cost of goods sold – variable (37,500 x 80%) /12×2 and 3 (5,000) (7,500) Variable cost of finished stock 30,000 / 24,000 × 1,000 (1,250) - Variable operating expenses (105 × 3 × 2,000) (630) (630) Payment of fixed costs (457 × 2.5) / (457 × 3.0) (1,143) (1 ,372) (59,992) 2,438

Month 1st Month Qtr. 2nd Qtr. 1 2 3 4 5 6 ---------- Rs. in ‘000 ----------

Working for credit sales Credit sales (18,000÷12×2,200) 3,300 3,300 3,300 3,300 3,300 3,300 Settlement – 70% 2,310 2,310 2,310 2,310 2,310 28% 924 924 924 924 Gross receipts 2,310 3,234 5,544 3,234 3,234 3,234 9,702 Tax @ 6% (333) (582) Receipts net of tax 5,211 9,120

MANAGEMENT ACCOUNTING

Suggested Answers Final Examinations – Summer 2010

Omair Jamal Page 3 of 7

Operating expenses Total operating expenses – given 4,800 Less: Variable cost per unit (105 × 24,000) (2,520) Bad debt expense (2,200 × 18,000 × 2%) (792) Fixed operating expenses 1,488 Fixed cost Fixed factory overheads 7,500 Less: Depreciation (60m – 7.5m) / 15 (3,500) Fixed operating overheads 1,488 5,488 Fixed cost per month 457 A.4 Noureen Industries Limited

Increased

commission 20% Contribution Margin Own sales

department ------------Rs. in ‘000s----------- Sales 80,000 80,000 Less: Variable expenses Manufacturing costs 44,800 44,800 Sales commission 16,000 4,000 Finance cost 150 150 60,950 48,950 Contribution margin 19,050 31,050 Contribution margin – as % of sales 23.8 38.8 Fixed expenses Fixed overheads 6,500 6,500 Depreciation 700 700 Fixed admin costs 2,200 2,200 Finance cost 600 600 Fixed marketing costs 7,000 10,000 17,000 Equal net income level: Let the required sales level be x. Net operating income with increased commission = 0.238x – 10,000 Net operating income with own sales force = 0.388x – 17,000 Both will be equal at: 0.388x – 17,000 = 0.238x – 10,000 0.15x = 7,000 x = 46,667 It would be beneficial for NIL to establish a full-fledged sales department if sales exceed Rs. 46,667,000. A.5 Manufacturing cost of product B Material A (105,000 × 250) 26,250,000 Material B (120,000 × 60) 7,200,000 (90,000× 70) 6,300,000 13,500,000 Labour (W-1) [60 × 200 × 6 ×448 (W-1)] 32,256,000 Variable factory overhead [200×6×448(W-1) hours×8] 4,300,800 Fixed factory overhead (W–2) 4,425,000 Manufacturing cost of product B 80,731,800

MANAGEMENT ACCOUNTING

Suggested Answers Final Examinations – Summer 2010

Omair Jamal Page 4 of 7

W-1: Labour Total working hours per labour (200 × 6) 1,200 Units produced in first 300 hours (300/6) 50 Units produced in remaining 900 hours (900/5) 180 No. of units produced per worker 230 No. of full units 100,000 Normal loss (5,000 × 60%) 3,000 103,000 No. of workers required (103,000 / 230) 448 W-2: Fixed overheads Savings at 70% capacity for 6 months [6/12 × (45.0 – 33.75)m] 5,625,000 Contribution from C - 10% capacity (70 – 60) for 6 months 1,200,000 A 6,825,000 Contribution from C - 40% capacity for 6 months B 4,800,000 Higher of A or B above 6,825,000 Less: contribution from C @ 20% capacity for 6 months 2,400,000 Opportunity cost of utilizing 20% capacity 4,425,000 A.6 (a) Most profitable option Costs Product AMY at

Faisalabad Product BNZ at

Lahore Total Cost of 5,000 units 2,760,000 1,585,000 Total Cost of 1,500 units A 870,000 535,000 Variable cost of 3,500 units 1,890,000 1,050,000 Variable cost per unit 540 300 Variable cost for 1500 units B 810,000 450,000 Fixed cost A –B 60,000 85,000 Option-1 If Lahore Factory purchases XPY from market for production of BNZ, contribution margin

from sale of BNZ could be as follows: BNZ output

(units) Revenue Variable cost excluding XPY Cost of XPY Contribution margin

(1) (2) (3) (1)-(2)-(3) 1,500 1,800,000 450,000 1,087,500 262,500 3,000 3,480,000 900,000 2,175,000 405,000 3,500 3,920,000 1,050,000 2,537,500 332,500 4,000 4,320,000 1,200,000 2,900,000 220,000 5,000 5,150,000 1,500,000 3,625,000 25,000

MANAGEMENT ACCOUNTING

Suggested Answers Final Examinations – Summer 2010

Omair Jamal Page 5 of 7

In addition, contribution margin earned from AMY produced by Faisalabad and sold outside could be as follows:

Product AMY output (units) Revenue Variable cost Contribution margin

1,500 1,275,000 810,000 465,000 3,000 2,475,000 1,620,000 855,000 3,500 2,800,000 1,890,000 910,000 4,000 3,100,000 2,160,000 940,000 5,000 3,500,000 2,700,000 800,000 Maximum Contribution Margin Contribution margin from production of 3,000 units of BNZ by Lahore 405,000 Contribution margin from sale of 4,000 units in Faisalabad 940,000 Total contribution margin 1,345,000 Option-2 If Lahore Factory uses AMY produced by Faisalabad factory contribution margin from sale of

BNZ could be as follows: BNZ output

(units) Revenue Variable cost excluding AMY Cost of AMY Contribution

margin (1) (2) (3) (1)-(2)-(3) 1,500 1,800,000 450,000 810,000 540,000 3,000 3,480,000 900,000 1,620,000 960,000 3,500 3,920,000 1,050,000 1,890,000 980,000 4,000 4,320,000 1,200,000 2,160,000 960,000 5,000 5,150,000 1,500,000 2,700,000 950,000 CM from in house production of 3,500 units 980,000 CM on production of 1,500 units in Faisalabad 465,000 1,445,000 To optimize profit, Lahore factory should use AMY produced by Faisalabad. (b) Minimum price acceptable to Faisalabad Factory Maximum contribution that could be earned by selling outside 940,000 Contribution earned by selling 1,500 units 465,000 Contribution to be earned by selling 3,500 units to Lahore 475,000 Profit to be earned per unit - 475000/3500 135.71 Variable cost per unit of AMY 540.00 Minimum price Faisalabad should charge for AMY 675.71 Maximum price acceptable to Lahore 725 A.6 (a) Most profitable option Costs Product AMY at

Faisalabad Marks

Product BNZ at Lahore

Total Cost of 5,000 units 2,760,000 1,585,000 Total Cost of 1,500 units A 870,000 535,000 Variable cost of 3,500 units 1,890,000 1,050,000 Variable cost per unit 540 300 Variable cost for 1500 units B 810,000 450,000 Fixed cost A –B 60,000 85,000

MANAGEMENT ACCOUNTING

Suggested Answers Final Examinations – Summer 2010

Omair Jamal Page 6 of 7

Option-1: Contribution margin earned from AMY produced by Faisalabad and sold outside could

be as follows: Product AMY output

(units) Revenue Variable cost Contribution margin

1,500 1,275,000 810,000 465,000 3,000 2,475,000 1,620,000 855,000 3,500 2,800,000 1,890,000 910,000 4,000 3,100,000 2,160,000 940,000 5,000 3,500,000 2,700,000 800,000 If Lahore Factory purchases 1,000 units of AMY from Faisalabad and balance units of XPY form

market for production of BNZ contribution margin from sale of BNZ could be as follows:

BNZ output (units) Revenue

Variable cost

excluding XPY

Cost of AMY (1,000 units only)

Cost of XPY (balance

units) Contribution margin

(1) (2) (3) (4) (1)-(2)-(3)-(4) 1,500 1,800,000 450,000 540,000 362,500 447,500 3,000 3,480,000 900,000 540,000 1,450,000 590,000 3,500 3,920,000 1,050,000 540,000 1,812,500 517,500 4,000 4,320,000 1,200,000 540,000 2,175,000 405,000 5,000 5,150,000 1,500,000 540,000 2,900,000 210,000 Maximum Contribution Margin Contribution margin from sale of 4,000 units in Faisalabad 940,000 Contribution margin from production of 3,000 units of BNZ by Lahore 590,000 Total contribution margin 1,530,000 Option-2 If Lahore Factory uses AMY produced by Faisalabad factory contribution margin from sale of

BNZ could be as follows: BNZ output

(units) Revenue Variable cost excluding AMY Cost of AMY Contribution

margin (1) (2) (3) (1)-(2)-(3) 1,500 1,800,000 450,000 810,000 540,000 3,000 3,480,000 900,000 1,620,000 960,000 3,500 3,920,000 1,050,000 1,890,000 980,000 4,000 4,320,000 1,200,000 2,160,000 960,000 5,000 5,150,000 1,500,000 2,700,000 950,000 CM from in house production of 3,500 units 980,000 CM on production of 1,500 units in Faisalabad 465,000 1,445,000 To optimize profit, Faisalabad factory should sell 4,000 units outside and 1,000 units to Lahore factory at Rs.

540 per unit and Lahore factory should purchase balance units of XPY from market.

MANAGEMENT ACCOUNTING

Suggested Answers Final Examinations – Summer 2010

Omair Jamal Page 7 of 7

(b) Minimum price acceptable to Faisalabad Factory Maximum contribution that could be earned by selling outside 940,000 Contribution earned by selling 1,500 units 465,000 Contribution to be earned by selling 3,500 units to Lahore 475,000 Profit to be earned per unit - 475000/3500 135.71 Variable cost per unit of AMY 540.00 Minimum price Faisalabad should charge for AMY 675.71 Maximum price acceptable to Lahore 725

(THE END)

MANAGEMENT ACCOUNTING Suggested Answers

Final Examinations – Winter 2009

A.1 Budgeted production Budget production is 70% of the designed capacity

(150 tons × 70% × 24 hours × 30 days × 12 months) 907,200 tons (A) Raw material Quantity of raw material required

(1.25 tons × 907,200 tons of finished product) 1,134,000 tons Quantity of raw material for each shipment

(1,134,000 tons ÷ 12 of finished product) 94,500 tons Total cost of purchases including transportation and other variable purchase

cost for each ton of product (Rs. 2,500 × 62.4%) Rs. 1,560/ton

Per ton FOB price of raw material (Rs. 1,560 × 100 ÷ 130) ÷ 1.25 Rs. 960 Total amount to be paid to supplier for each shipment (Rs. 960 × 94,500 tons) Rs. 90.72 mln Credit period (45-30 days) 15 days Trade credit: Average amount of liability (Rs. 90.72 million × 15/30) Rs. 45.36 mln Cost of consumables, spares and processing per ton (2,500 × 37.6%) Rs. 940

(B) Inventory Rs. in 000’ Raw Material (94,500 tons ÷ 2 × Rs. 960 ×1.3) 58,968 Work in progress (1,000 × Rs. 960 × 1.3) + (1,000 × 940 × 50%) 1,718 Finished products (907,200 × 15 ÷ 30 ÷ 12) × Rs. 2,500 94,500 Spares & consumables 20,000 175,186 (C) Debtors Rupees Corporate clients (40% × 945,000 tons × 10 ÷ 360 × Rs. 3,300 ×1.02) 35,343,000 Individual clients (30% × 945,000 tons × 2 ÷ 360 × Rs. 3,300) (5,197,500) Export clients (30% × 945,000 tons × 5 ÷ 360 × Rs. 3,300 ×1.05) 13,643,438 43,788,938 Budgeted Sales quantity: Tons Production during the year 907,200 Opening inventory – 1/12 of above 75,600 Closing inventory – 1/24 of the above (37,800) Budgeted sales 945,000 Budgeted Price Variable cost 2,500 Fixed Cost (Rs. 10.584 million ×12 ÷ 907,200 tons) 140 Total cost 2,640 Gross profit at 20% of selling price 660 Sales Price 3,300 (D) Other credit Fixed cost (Rs. 10.584 million × 15 / 30 days) 5,292,000 Other variable cost: 940 (Rs. 2,500 – Rs. 1,560) × 907,200 × 15 /360 days 35,532,000 40,824,000

MANAGEMENT ACCOUNTING Suggested Answers

Final Examinations – Winter 2009

Working Capital requirement Average value of debtors 43,788,938 Average value of inventory 175,186,000 Average value of trade credit (45,360,000) Average value of other credit (40,824,000) 132,790,938

A.2

Units Average time

Cumulative time

1 40.00 40

2 38.00 76

4 36.10 144

8 34.30 274

16 32.58 521

32 30.95 990

64 29.40 1,882

No. of workers

Available hours*

Average time per unit

Production per worker

Total production

No. of Hours

40 522 32.58 16 640 20,880

7 992 30.95 32 224 6,944

153 1,882 29.40 64 9,792 287,946

153 206 28.00 7 1,071 31,518

11,727 347,288

Available hours: Up to March 31 174 x 3 522

Up to June 21 174 x 5.7 992

Up to December 31 174 x 12 [1,882+206] 2,088

Cost of production

Units Rate Total cost

Materials 11,727 10,000 117,270,000

Labour 347,288 110 38,201,680

Overheads 11,727 4,000 46,908,000

202,379,680

Production (units)

11,727

Average cost per unit

17,258

Selling price per unit

21,573 Labour cost includes 10% bonus

MANAGEMENT ACCOUNTING Suggested Answers

Final Examinations – Winter 2009

A.3 Standard weight of one unit of finished goods 11.88 kgs

Total input of raw material required for one unit of finished product {11.88 ÷ (100% – 12%)} 13.50kgs

Standard material input: A:6.75kgs, B:4.50kgs, C:2.25kgs

Material A B C

A Year-end inventory Given kgs 1,014,200 754,000 228,000

B Ratio of inventory to material consumed Given % 11 13 8

C Material consumed A/B kgs 9,220,000 5,800,000 2,850,000

D Purchases during the year A+C kgs 10,234,200 6,554,000 3,078,000

E Purchases during :

Oct-Mar D/2 kgs 5,117,100 3,277,000 1,539,000

Apr-Sept D/2 kgs 5,117,100 3,277,000 1,539,000

F Value of year-end inventory Given Rs. 6,744,430 3,883,100 1,390,800

G Actual price per kg

Oct-Mar F/A Rs. 6.65 5.15 6.10

H Apr-Sept G/1.1 Rs. 6.05 4.68 5.55

J Average price Oct-Sept (G+H) / 2 Rs. 6.35 4.92 5.83

K Purchases during year J*D Rs. 64,987,170 32,245,680 17,944,740

L Material consumed at actual price K-F Rs. 58,242,740 28,362,580 16,553,940

M Standard price Given kg 6.40 4.85 5.90

N Standard cost C x M Rs. 59,008,000 28,130,000 16,815,000

P Price variance favourable/(unfavourable) L – N Rs. 765,260 (232,580) 261,060

Total price variance – Favourable

793,740

Mix variance

: Raw

material

Actual quantity used

(kgs)

Standard mix of actual quantity used

Actual Variances

(kgs)

Standard price per

kg Variances (rupees)

Ratio kgs A 9,220,000 3/6 8,935,000 (285,000) 6.40 (1,824,000) B 5,800,000 2/6 5,956,667 156,667 4.85 759,835 C 2,850,000 1/6 2,978,333 128,333 5.90 757,165 17,870,000 17,870,000 NIL (307,000) Yield Variance

: Raw

material

Standard mix of actual

quantity used

Standard usage for actual output (kgs)

Variances (kgs)

Standard price per

kg

Variances Rs.

A 8,935,000 8,910,000 (25,000) 6.40 (160,000) B 5,956,667 5,940,000 (16,667) 4.85 (80,835) C 2,978,333 2,970,000 (8,333) 5.90 (49,165) 17,870,000 *17,820,000 (50,000) (290,000) {Output 1.32 million units x standard input per unit 13.50 kgs(6.75 + 4.50 + 2.25kgs)} Material usage variance (597,000) : Mix + yield variances

MANAGEMENT ACCOUNTING Suggested Answers

Final Examinations – Winter 2009

A.4 Cost of components without inspection:

Y Z

Total good components required (A) 10,000 10,000

Defectives expected (7/93 and 11/89 of 10,000) (B) 753 1,236

Total components to be purchased A + B = (C) 10,753 11,236

COSTS:

Purchase price of components @ 90 x (C) and 87 x (C) 967,770 977,532

Production cost of defective units:

Material cost at start - 50% of 420 × (B) 158,130 259,560

Balance processing costs {B*60% of (720-210)} 230,418 378,216

Sale proceeds of defectives (B) x 200 (150,600) (247,200)

Total cost of components (including defective components and defective units produced) 1,205,718 1,368,108

Cost of components with inspection: Y Z

Total good components required (D) 10,000 10,000

Defectives expected B ÷ 10 (E) 75 124

Total components required D + E (F) 10,075 10,124

COSTS:

Purchase price of components @ 90 x (F) and 87 x (F) 906,750 880,788

Production cost of defective units:

Material cost at start - 50% of 420 x (E) 15,750 26,040

Balance processing costs (E) * 60% of (720 -210) 22,950 37,944

Sale proceeds of defectives (E) x 200 (15,000) (24,800)

Inspection cost @ Rs. 20 per component {20 x (C)} 215,060 224,720

Total cost of components (including defective components and defective units produced) 1,145,510 1,144,692

Conclusion: The best option is that company should buy component Z and should carry out the inspection.

A.5 Option 1: Manufacturing all units at own factory 350,000 units 50,000 units 400,000 units Rate Amount Rate Amount Amount Rs. in ‘000’ Rs. in ‘000’ Rs. in ‘000’ Material – units 24,000 8,400,000 24,000 1,200,000 Labour 3,400 1,190,000 6,800 340,000 Overheads 1,680 588,000 1,680 84,000 10,178,000 1,624,000 Existing fixed cost (260,000 x 1,120) 291,200 Less: Cost of idle labour (260,000 x 3,400 x 0.15/0.85 x 90%) (140,400) 150,800 Additional fixed costs (10% of 150,800) 15,080 Discount on material 2.5% of 9.6(8.4+1.2) billion (240,000) Cost of producing 350,000/50,000/400,000 units 10,328,800 1,399,080 11,728,880

MANAGEMENT ACCOUNTING Suggested Answers

Final Examinations – Winter 2009

Option – 2 Produce 350,000 units locally and import 50,000 units from Italy

Rs. in ‘000’

Production of 350,000 units 10,328,800 Purchase of 50,000 units from outside @ 29,000 1,450,000 Total cost for 400,000 units 11,778,800

Option 3 Impact all (400,000) units from Italy Purchase of 4000,000 units from outside @ 29,000 11,600,000

Add: fixed cost 150,800 Total cost 11,750,800

Decision The company should produce 400,000 units at its own manufacturing facility.

A.6 PRICING OF NEW PRODUCTS

Calculation of expected sale

Pack size Total 500 grams 1 kg 2 kg

Units (a) 200,000 120,000 90,000

Total production (Kgs.) (b) 400,000 100,000 120,000 180,000

Percentage of total production 100% 25% 30% 45%

Consumption of Material A (Kgs) (c) 200,000 50,000 60,000 90,000

Cost of Material A {300 × (c)} (d) 60,000,000 15,000,000 18,000,000 27,000,000

Material B {118.125(W-1) × (a)} (e) 47,250,000 11,812,500 14,175,000 21,262,500

Packaging cost {(a) x 30, 40; 55} (f) 15,750,000 6,000,000 4,800,000 4,950,000

Labour {7.5625 (W-3) × (b)} 3,025,000 756,250 907,500 1,361,250

Fixed overheads {9.375 (W-4) x (b)} 3,750,000 937,500 1,125,000 1,687,500

Total cost 129,775,000 34,506,250 39,007,500 56,261,250

Sales (cost + 25%) 162,218,750 43,132,813 48,759,375 70,326,563

Sale price / unit

216 406 781

W-1 Material B Qty Rate Amount

Opportunity cost of 100,000 kgs (W-2) 100,000

12,000,000

Current disposal price of remaining available material 150,000 110 16,500,000

Purchase price of additional requirement 150,000 125 18,750,000

400,000 118.125 47,250,000

W-2 Opportunity cost of 100,000 kgs

Sale Price 100,000 160 16,000,000

Less: additional cost

(4,000,000)

12,000,000

Sale price if sold without processing

11,000,000

Higher of the above

12,000,000

W-3 Labour

Skilled Labour [(400,000 / 100 × 5) × 70 1,400,000

Unskilled Labour (400,000 / 100 × 10) × 45 1,800,000

Less: Skilled Labour - Idle hours now saved (5,000 × 70 /2) (175,000)

3,025,000

Cost per Kg 7.5625

W-4 Current fixed expenses 25,000,000 × (100-25)% = Rs. 18,750,000

Production including new product (2,000,000×50%)kgs + 400,000 kgs = 1,400,000 Kgs.

Capacity utilization after introduction of new product = 70%

MANAGEMENT ACCOUNTING Suggested Answers

Final Examinations – Winter 2009

Fixed expenses “ “ “ “ “ (25,000,000×90%) = 22,500,000

Additional fixed expenses on a/c of new product Rs. 3,750,000

Cost per Kg (for allocation purpose) Rs. 9.375 (The End)

MANAGEMENT ACCOUNTING Suggested Answer

Final Examinations – Summer 2009

Page 1 of 6

Ans.1 ABC LIMITED Actual Jan-May 2009 Rupees Sales (105,000x350) 36,750,000 Variable costs: Raw materials (105,000x90) (9,450,000) Direct labor (300 × 0.4) x 105,000 (12,600,000) Other variable costs (300-112.50-120) x 105,000 (7,087,500) Contribution margin 7,612,500 Revised Plan Jun-Dec 2009 LGV HGV Total Sale price per unit 270 385.00 Variable cost: Raw material cost A (25x2x5/8) (31.25) B (45x3x3/8) (45 × 3 × 2)/8 (33.75) (65.00) (90.00) Direct labor cost (300×0.4) (120.00) (120 × 0.6 × 1.1) (79.20) Factory overhead cost (300-112.5-120) (67.50) (67.5 × 0.9) (60.75) Total variable cost (204.95) (277.50) Contribution margin Rs 65.05 107.50 Sales mix ratio 2 1 3 Aggregate contribution margin Rs. 130.10 107.50 237.60 Fixed cost Jan-Dec: Fixed cost for the year 25,000,000 Additional marketing cost 3,000,000 10% depreciation on machine cost Jun-Dec 2009 70,000 28,070,000 Contribution recovered Jan-May 2009 (7,612,500) Required contribution for Jun-Dec 2009 20,457,500 Break even Sale quantity Jun-Dec 2009: Break even quantity for:

High grade (20,457,500/237.60)

86,101

Low grade (86,101 × 2) 172,202 Break even Sale amount Jun-Dec 2009 Rs. 46,494,540 33,148,885 79,643,425

MANAGEMENT ACCOUNTING Suggested Answer

Final Examinations – Summer 2009

Page 2 of 6

Ans.2 Sales volume margin/profit/contribution variance = 7,125,000 / 50,000 × 1,500 = Rs. 213,750 (Fav) (W–1) Sales Price Variance = 51,500 units (Rs. 540 – Rs. 530) = Rs. 515,000 (Adv) (W–2, 3, 4) Raw material Price variance = (Rs. 53 -50) × 159,650 = Rs. 478,950 (Adv) (W–6, 7, 8) Raw material quantity variance = {159,650 – (51,500 × 3)} × 50 = Rs. 257,500 (Adv) (W–6, 8, 9) Labour rate Variance = (Rs 75 – Rs. 70) × 51,500 × 2.625 hours = Rs. 675,937.50 (Fav) (W–9, 10, 11, 12, 13) Labour efficiency variance = 1/8 hour × Rs. 75 × 51,500 = Rs. 482,812.50 (Adv) (W–2, 12) Variable overhead efficiency variance = 1/8 hour × (24x51,500) = 154,500 (Adv) (W–14) Variable overhead spending / expenditure variance (24 – 22) × 51,500 × 2.625 = Rs. 270,375 (Fav) (W–14, 15) W-1: Budgeted Sales quantity: 1,500 / 0.03 = 50,000 units W-2: Actual Sales quantity 50,000 + 1,500 = 51,500 units W-3: Budgeted sale price: 27,000,000 / 50,000 = Rs. 540 per unit W-4: Actual sale price: 540 – 10 = Rs. 530 per unit W-5: Budgeted raw material quantity = 50,000 units × 3 kgs = 150,000 kgs W-6: Budgeted material price = 7,500,000 ÷ 150,000 kgs = Rs. 50 per kg (W–5) W-7: Actual material price = Rs. 50 × 1.06 = Rs. 53 per kg W-8: Total actual quantity used = Rs. 8,461,450 ÷ Rs. 53 = 159,650 kgs W-9: Budgeted labour cost per finished unit = 9,375,000 ÷ 50,000 = Rs. 187.50 W-10: Budgeted labour time for one finished unit = [(Rs. 187.5) ÷ (Rs 50 × 150%)] = 2.5 hours (W–10) W-11: Actual labour time taken for one finished unit = 2.5 + (1÷ 8) = 2.625 hours

MANAGEMENT ACCOUNTING Suggested Answer

Final Examinations – Summer 2009

Page 3 of 6

W-12: Budgeted labour cost per hour = (Rs. 187.5 ÷ 2.50 hours) = Rs. 75 per hour W-13: Actual labour cost per hour = (Rs. 9,463,125 ÷ (2.625 hours × 51,500) = Rs. 70 per hour W-14: Budgeted variable overhead rate per hour 3,000,000 / (50,000 × 2.50) = Rs. 24 per labour hour W-15: Actual variable overhead rate per hour 2,974,125 / (2.625 × 51,500) = Rs. 22 per labour hour RECONCILIATION OF BUDGETED CONTRIBUTION AND ACTUAL CONTRIBUTION Rupees

Budgeted profit 7,125,000 Sales volume margin variance 213,750 Sale price variance (515,000) Material price variance (478,950) Material quantity (usage) variance (257,500) Labour rate variance 675,937.50 Labour efficiency variance (482,812.50) Variable overhead efficiency variance (154,500) Variable overhead spending / expenditure variance 270,375 Actual profit 6,396,300

Patient days of

occupancy

Diff from Average (x)

Cost of Supplies Rs. ‘000’

Diff from Average (y)

Col 2 sqrd Σx2

(2) x (4) Rs. ‘000’ Σxy

Ans.3

1 2 3 4 5 6 Dec *8,370 -120 1,665 -55.0 14,400 6,600 Jan 8,649 159 1,804 84.0 25,281 13,356 Feb 8,232 -258 1,717 -3.0 66,564 774 Mar 8,742 252 1,735 15.0 63,504 3,780 Apr 7,740 -750 1,597 -123.0 562,500 92,250 May 9,207 717 1,802 82.0 514,089 58,794 Total 50,940 0 10,320 1,246,338 175,554 Average 8,490 1,720 *8370 = 300 × 90% × 31 days Variable expenses = Σxy / Σx2 Col 6 / Col 5 = 175,554 / 1,246,338 = 0.14086 = 0.14086 × 1000 = Rs. 140.86 Variable rate per patient per day

MANAGEMENT ACCOUNTING Suggested Answer

Final Examinations – Summer 2009

Page 4 of 6

8

Alternate answer of Question No.3

Patient days of occupancy

Cost of supplies 1 x 2 Col 1 squared

1 (x) 2 (y) 3 (x*y) 4 (x2) Dec 8,370 1,665 13,936,050 70,056,900 Jan 8,649 1,804 15,602,796 74,805,201 Feb 8,232 1,717 14,134,344 67,765,824 Mar 8,742 1,735 15,167,370 76,422,564 Apr 7,740 1,597 12,360,780 59,907,600 May 9,207 1,802 16,591,014 84,768,849 Total 50,940 10,320 87,792,354 433,726,938 Variable Cost b

10.14085585

7,478,0281,053,324

50,940)*(50,940-8433,726,93*610,320)*(50,940-87,792,354*6

x)(xn

y)x)((xyn22 ===

−=

∑∑∑ ∑ ∑

Variable cost = 1,000 × 10.14085585 = Rs. 140.86 per day per patient Ans.4 (a)

Activity Time in days EST EFT LST LFT Total

Float* 0 – 1 2 0 2 0 2 0 1 – 2 8 2 10 8 16 6 1 – 3 10 2 12 2 12 0 2 – 4 6 10 16 16 22 6 2 – 5 3 10 13 22 25 12 3 – 4 3 12 15 19 22 7 3 – 6 7 12 19 12 19 0 4 – 7 5 16 21 22 27 6 5 – 7 2 13 15 25 27 12 6 – 7 8 19 27 19 27 0 * LFT-EST-Time in days (b) The critical path is 0 – 1 – 3 – 6 – 7. The project duration is 27 days.

10

7

8

2

5 2

5

0 1 4

2

7

3 6

3

3

6

MANAGEMENT ACCOUNTING Suggested Answer

Final Examinations – Summer 2009

Page 5 of 6

Ans.5 Computation of budgeted gross profit based on: Existing

budget Rupees

Budget based on

recommended planRupees

Material A (2 M kgs x 70% × 32) / 0.92 48,695,652.16 (2 M kgs × 70% x 32) / 0.984 45,528,455 Material B (2M x 30% × 10) / 0.92 6,521,739.13 (2M × 30% × 10) / 0.984 6,097,561 Inspection cost (2M × 0.50) / 0.984 1,016,260.00 Labour Cost (15 /60 × 2M × Rs. 400/8) 25,000,000.00 (Rs.25m-Rs.3m (Note) 22,000,000.00 Variable overhead (2 M × Rs. 5)/0.92 10,869,565.21 (Rs.5 x 80%) × [(2M / (1-0.016)] 8,130,081.30 Fixed Overhead 4,000,000.00 (Rs. 4,000,000–25%) 3,000,000.00 95,086,956.50 85,772,357.30 Savings 9,314,599.20 (Note) Savings in Labour Cost: Average labour time for industry (15 minutes /1.25) 12 Minutes Benefits of time saving [(15 minutes – 12 minutes) /60] × 2 M × 400/8 Rs. 5,000,000 Workers share (Rs. 5 million × 40%) Rs. 2,000,000 Savings Rs. 3,000,000

Ans.6 Option I: Cost of short term loan per month: Rate 18% per annum = 1.5% per month Cost of funds for 6 months = {10,000,000 × (1.015) 6}-10,000,000}= Rs. 934,433 Cost of funds for 1 month = 934,433 / 6 = 155,739 Option II: Cost of financing through supplies: Opportunity cost per month = 200,000 / 9,800,000 = 2.04% / 2 = 1.02% or Rs. 102,041 per month Option III: Cost of factoring per month: Rupees Credit Sales 25,000,000 × 60/100 15,000,000 Interest charges 15,000,000 × 45/30 x 75% × 1.25% 210,938 Fee 15,000,000 × 2% 300,000 Total Charges 510,938 Less : Savings in Bad debts and cost of credit control 200,000 Financial charges saving 63,641* (263,641) 247,297 Cost of funds = Rs. 303,547 per month * Advance 75% of 15 million x 45/30 16,875,000 Less: interest charges (210,938) Factors fees (300,000) 16,364,062 Less: requirement (10,000,000) Overdraft reduction 6,364,062 Interest at 1% per month 63,641 Conclusion:

Option II is the cheapest option. The company should forego the cash discount of 2% and avail

credit for further 60 days.

MANAGEMENT ACCOUNTING Suggested Answer

Final Examinations – Summer 2009

Page 6 of 6

Ans.7 (a) X Y Z Total Number of units A 10,000 2,000 50,000 Direct labour hours per unit B 2.5 5.0 2.8 Direct labour hours (A× B) C 25,000 10,000 140,000 175,000

Total factory overheads D 1,400,000 Factory overhead rate per hour (D/C) E Rs. 8

Cost per unit - single factory overhead rate

method (B × E) F 20 40 22.40 (b) Activity based costing Set-up costs Batch size G 125 50 10,000 Set-ups (A ÷ G) H 80 40 5 125 Set-up costs J 274,400 137,200 17,150 428,750 Production control Machine hours per unit K 7.5 10.0 3.0 Total machine hours (A × K) L 75,000 20,000 150,000 245,000 Production control M 75,000 20,000 150,000 245,000 Quality control Allocation No. of inspections N 5% 5% 2% Units inspected (A × N) P 500 100 1,000 Hours per unit inspected Q 0.2 0.5 0.1 Total inspection hours (P × Q) R 100 50 100 250 Quality control costs S 73,500 36,750 73,500 183,750 Materials management No. of requisitions T 320 400 30 750 Material management costs U 156,800 196,000 14,700 367,500 Factory overheads – General Allocated on the basis of direct labour hours V 25,000 10,000 140,000 175,000 Total cost (J+M+S+U+V) W 604,700 399,950 395,350 1,400,000

Factory overhead cost per unit - activity

based costing (W ÷ A) Rs. 60.47 199.98 7.91

(The End)

MANAGEMENT ACCOUNTING Suggested Answers

Final Examinations – Winter 2008

Page 1 of 7

Ans.1 (a) Material Labour Variable

overheads

Total Variable

Cost

Fixed overheads Total Cost

Equivalent units:

Completed units (6,000 + 1,000 – 800) 6,200 6,200 6,200 6,200

Closing work-in-progress 1,200 600 600 600 Opening work-in-progress (500) (200) (200) (200) Total equivalent units 6,900 6,600 6,600 6,600 Total cost (Rs.) 83,490,000 14,256,000 10,890,000 108,636,000 17,490,000 126,126,000 Cost per unit (Rs.) 12,100 2,160 1,650 15,910 2,650 18,560

b

(i) Absorption costing profit statement: Rupees Sales (6,000 × 24,000) 144,000,000 Op WIP 6,700,000 Op finished goods (17,000 × 800) 13,600,000 Production cost 126,126,000 Closing WIP (18,396,000) Closing finished goods stock (18,560 × 1,000) (18,560,000) 109,470,000 Gross profit 34,530,000 Less: variable selling and administration costs (1,600 ×

6,000) 9,600,000 Fixed selling and administration costs 12,000,000 21,600,000 Net profit 12,930,000 (ii) Marginal costing profit statement: Rupees Sales 144,000,000 Opening WIP 6,200,000 Opening finished goods (800 x 14,500) 11,600,000 Variable cost of production 108,636,000 Closing WIP (16,806,000) Closing finished goods stock (1,000 x 15,910) (15,910,000) Variable cost of sales 93,720,000 Variable selling and administration costs (1,600 × 6,000) 9,600,000 103,320,000 Contribution 40,680,000 Less: Fixed costs (17,490 + 12,000) 29,490,000 Net profit 11,190,000 Working Closing work-in-progress (Rs.) 14,520,000 1,296,000 990,000 16,806,000 1,590,000 18,396,000 Cost per unit last year 11,000 2,000 1,500 14,500 2,500 17,000 Opening work-in-progress (Rs.) 5,500,000 400,000 300,000 6,200,000 500,000 6,700,000

MANAGEMENT ACCOUNTING Suggested Answers

Final Examinations – Winter 2008

Page 2 of 7

Ans.2 (a) ROI = net profit / total assets (investment) Computation of net profit Rs. in million Annual profit before depreciation and financial charges 150 Depreciation [(Rs. 500 M - 20 M) / 10 years] (48) Financial Charges (Rs. 600 × 70% × 10%) (42) 60 Computation of net capital employed (mid year) for year 2009 Rs. in million Net Book Value at 1st January, 2009 500 Net Book Value at 31st December, 2009 [(480 × 9/10) + 20] 452 Mid Year Value for year 2009 [(500 + 452) /2] 476 Working Capital 100 Average net capital employed 576 ROI for the year 2009 [(Rs. 60M / Rs. 576M) × 100] 10.42% Computation of average net capital employed (mid year) for year 2015 Rs. in million Net Book Value at 1st January, 2015 [(480 × 4/10) + 20] 212 Net Book Value at 31st December, 2015 [(480 × 3/10) + 20] 164 Mid Year Value for year 2015 [(212 + 164) /2] 188 Working Capital 100 Average net capital employed 288 ROI for the year 2015 [(Rs. 60 / Rs. 288) × 100] 20.83% (b) Comments on appropriateness of the result

1. ROI method focuses on short term performance whereas investment decision should be

evaluated on the life of the project. 2. Although the net profit for the years 2009 & 2015 are same but the ROI is much higher in

2015 as compared to 2009 which shows that it is not an appropriate ratio for comparing the performance on year to year basis.

Ans.3 Total Production Capacity Kgs Model A (7,500 x 80) 600,000 Model Z (7,500 x 100) 750,000 Computation of per kg cost (Model A) Per Unit Cost Total Cost Rupees Rupees Raw Material Cost (400 / 0.85) 470.59 282,354,000 Natural Gas (0.5 MMBTU × Rs. 80) 40.00 24,000,000 Electricity (2 KWH × 12 Rs.) 24.00 14,400,000 Water (5 gallons × Rs. 2) 10.00 6,000,000 Plant depreciation (33,000,000 / 600,000) 55.00 33,000,000 Labour cost (30,000,000/600000) 50.00 30,000,000 Other production overhead (60,000,000 / 600) 100.00 60,000,000 749.59 449,754,000

MANAGEMENT ACCOUNTING Suggested Answers

Final Examinations – Winter 2008

Page 3 of 7

Computation of per kg cost (Model Z) Per Unit Cost Total Cost Rupees Rupees Raw Material Cost 400.00 300,000,000 Natural Gas (0.4 MMBTU × Rs. 80) 32.00 24,000,000 Electricity (1.5KWH × Rs. 12) 18.00 13,500,000 Water (4 gallons × Rs. 2) 8.00 6,000,000 Plant depreciation (37,500,000 / 750,000) 50.00 37,500,000 Labour Cost (33,000,000/600) 55.00 41,250,000 Other production overheads (80,500,000 / 750,000) 107.33 80,500,000 670.33 502,750,000 Wastage (10/90 x 670.33) 74.48 55,860,000 744.81 558,610,000 Other production overheads for Model Z Rupees Fixed cost (40% x 70.0 million) 28,000,000 Variable cost (70 million x 60% × 75 / 60) 52,500,000 80,500,000 Selling and administration expenses for Model Z Fixed cost (60% x 45 million) 27,000,000 Variable cost (45 million x 40% × 75 / 60) 22,500,000 49,500,000 Computation of financial charges Rs. in million Rs. in million Investment Size Plant Cost 660.000 750.000 Working capital 108.000 135.000 768.000 885.000 60% Debt 460.800 531.000 Annual Financial Charges @ 12% 55.296 63.720

Profitability Analysis of Model A and Model Z

Model A Model Z Rupees Rupees Sales @ Rs. 900/ Kg 540,000,000 675,000,000 Cost of goods sold (449,754,000) (558,610,000) Gross Profit 90,246,000 116,390,000 Admin and selling overheads (35,000,000) (49,500,000) Financial Charges (55,296,000) (63,720,000) Net Profit (50,000) 3,170,000 Tax @ 30% - (951,000) (50,000) 2,219,000 Equity (768 × 40%) 307,200,000 (885 × 40%) 354,000,000 Return on equity (0.02) %) 0.63% Model Z is to be preferred over Model A.

MANAGEMENT ACCOUNTING Suggested Answers

Final Examinations – Winter 2008

Page 4 of 7

Ans.4 (a)

Existing

Assets/Liabilities Proposed

level Rupees Rupees Debtors 360,000 x 160 57,600,000 360,000 x 160 x 1.25 x 2 144,000,000 Stocks 360,000 x 120 x 60% 25,920,000 360,000 x 120 x 60% x 1.25 32,400,000 Creditors 2/3 of above (17,280,000) 2/3 of above (21,600,000) Finished goods 360,000*120*2 86,400,000 360,000 x 120 x 2 x 1.25 108,000,000 152,640,000 262,800,000 Rupees

Increase in working capital (Rs. 262,800,000 – Rs. 152,640,000) 110,160,000

Cost of funds @ 16% of above 17,625,600

Profit margin on extra sales 360,000*0.25*40*12 43,200,000 Extra profits are more than 2.4 times the cost of funds; hence the proposed credit policy is feasible. (b) Cost of factoring per month Rupees Fee (8,000,000 x 80% x 2% 128,000 Commission (8,000,000 x 30/20 x 4%) 480,000 608,000 Less : Savings in management costs (600,000 / 12) (50,000) Savings on bad debts (8,000,000 x 30/20 x 1%) (120,000) 438,000 Cost of short term finance from bank, per month Rupees Interest (8,000,000 x 0.8 x 18% / 12 ) 96,000 Processing fee (3% x 8,000,000 x 80%) 192,000 288,000 Obtaining short term loan facility is less costly and hence a better option. Ans.5 Computation of labour hours required Assuming that the learning curve rate is x: 800 × 4 × x × x = 2312 x2 = 2312 / 3,200 x = 0.85 Batches Cumulative quantity Cumulative average

hours per unit Cumulative hours

1 40 20 800 2 80 17 1,360 4 160 14.45 2,312 8 320 12.28 3,930 16 640 10.44 6,682 Hence, additional hours for 480 units = 6,682 – 2,312 = 4,370 hours

MANAGEMENT ACCOUNTING Suggested Answers

Final Examinations – Winter 2008

Page 5 of 7

Labour hour rate: Rupees

600 normal hours + 200 overtime hours 800,000 600 + 200 x 2 800,000 1,000 hours 800,000 Hourly rate 800

Direct labour:

Hours Direct

labour cost Rupees

8 workers for 10 weeks for 40 hours 3,200 @ Rs. 800 per hour 2,560,0002 workers for 4 weeks for 40 hours 800 @ Rs. 800 per hour 256,000Overtime 370 @ Rs. 1,600 per hour 592,000 4,370 3,408,000

Incremental cost of producing 480 units: Amount in Rs Direct materials (480 × 10,000) 4,800,000 Direct labour 3,408,000 Variable overhead (4,370 × 500) 2,185,000 10,393,000 Cost per unit (10,393,000/480) 21,652 Hence, quotation can be accepted at Rs 25,000 per unit. Ans.6 Computation of Sales for 2008

A B

Normal B

Corporate Total Ratio of sale price 1.00 1.60 1.44 Actual sale Qty 5,400.00 2,880.00 720.00 Ratio of sale value 5,400.00 4,608.00 1,036.80 11,044.80 Sales value 2,700,000.00 2,304,000.00 518,400.00 5,522,400.00 A B Current year’s production (at 90 % capacity) 5,400.00 3,600.00 Production at full capacity 6,000.00 4,000.00 If only B is produced the company can produce 9,000 units (4,000 + 6,000 / 1.2). Required production of B in the next year = (2,880 x 1.3) + (2 x 720) = 3744 + 1440 = 5,184 units Remaining capacity can be utilised to produce 4,579 units of A [(9,000 - 5,184) x 1.2]. Computation of Sales for 2009 Rupees Sales of A (4,579 x 500) 2,289,500 Sales of B (5,184 x 800) 4,147,200 6,436,700 Discount to Corporate customer (1,440 × 800 × 15%) 172,800 6,263,900

MANAGEMENT ACCOUNTING Suggested Answers

Final Examinations – Winter 2008

Page 6 of 7

Consumption of Raw Material Kgs Consumption of raw material in 2008 (A: 5,400 x 2.4 / 0.96) 13,500.00 Consumption of raw material in 2008 (B: 3,600 x 2.4 / 0.90) 9,600.00 Total 23,100.00 Rupees Price per kg of raw material ( 2,310,000 / 23,100) 100.00 Total expected consumption in 2009 (A: 4,579 x 2.4 / 0.96) 11,447.50 Total expected consumption in 2009 (B: 5,184 x 2.4 / 0.90) 13,824.00 Total consumption for 2009 25,271.50 Average price for 2009 ((100 x 3) + (110 x 9)) / 12 107.50 Total cost of raw material for 2009 2,716,686.25 Computation of Direct Labour Hours Labour hours used in 2008 (A: 5,400 × 5) 27,000 Labour hours used in 2008 (B: 3,600 × 6) 21,600 48,600 Labour hours forecast for 2009 (A: 4,579 × 5) 22,895 Labour hours forecast for 2009 (B: 5,184 × 6) 31,104 53,999 Increase in labour hours 5,399 Labour cost for 2009 (1.15 x (777,600 x 53,999 / 48,600) Rs. 993,582

Production overheads for 2008 : Rupees Fixed overheads (40% x 630,000) 252,000.00 Variable overheads (630,000-252,000) 378,000.00 A B Total Ratio of variable overheads 1.00 2.00 Total units produced 5,400.00 3,600.00 Product (units) (K) 5,400.00 7,200.00 12,600.00 Total variable overheads (Rs.) (L) 162,000.00 216,000.00 378,000.00 Per unit variable overheads (Rs.) (L /K) 30.00 60.00

MANAGEMENT ACCOUNTING Suggested Answers

Final Examinations – Winter 2008

Page 7 of 7

Production overheads for 2009: A B Total Fixed overheads (1.05 x 252,000) (Rs.) 264,600.00 Per unit variable overheads (Rs.) 33.00 66.00 Total units 4,579 5,184 Total variable overheads (Rs.) 151,107.00 342,144.00 493,251.00 Total overheads (Rs.) 757,851.00 PROFIT FORECAST STATEMENT FOR 2009 Rupees Sales 6,263,900.00 Material 2,716,686.25 Labour 993,582.00 Overheads 757,851.00 4,468,119.25 Gross margin 1,795,780.75 Selling and administration expenses (800,000 x 1.1) + 250,000 1,130,000.00 665,780.75

(THE END)

MANAGEMENT ACCOUNTING Suggested Answer

Final Examinations – Summer 2008

Page 1 of 8

Ans.1 (a) The company has to supply minimum sales to the customer as follows:

Unit Rate Rs. X 40,000 900 36,000,000

Y 96,000 1,200 115,200,000

151,200,000

Further sales possible (200,000,000 – 151,200,00) 48,800,000

X Y Contribution per unit Rs. 260 170 Contribution per hour Rs. 2,080 1,020 Contribution % on sales 29 14

X contributes more than Y. Therefore, 48,800,000 / Rs.900 = 54,222 units of X should be produced. Check whether this level of production can be attained in available hours:

Units Hours X (40,000+ 54,222 ) 94,222 11,778 Y 96,000 16,000 27,778

Therefore, maximum contribution / profit will be as follows: X Y Total - Rs. Sales in unit 94,222 96,000 Contribution per unit 260 170 Total contribution 24,497,720 16,320,000 40,817,720 (b) Increase / (decrease) in profit if the loan is taken

Extra Sales of X if loan is taken (60 mln / 900) 66,667 units Production possible in remaining hours (6,222* x 8) 49,776 units Contribution on 49,776 units (49,776 x 260) Rs. 12,941,760 Bank charges on Rs.25 mln at 16% 4,000,000 Additional contribution if bank facility availed 8,941,760 *(34,000 – 27,778) Ans.2 Computation of working capital Rupees Debtors: Exports (D*30/360) (Working 1) 40,916,667 Local customers with 2% discount (F*0.98*10/360) (Working 2) 6,533,333 Local customers with 1% discount (E*0.99*20/360) (Working 2) 6,600,000 Local customers who do not avail discount (C-100,000,000-E-F-13180000)/12 23,818,333 Advance against raw material C (N x 15/360x0.5) (Working 3) 2,043,215

MANAGEMENT ACCOUNTING Suggested Answer

Final Examinations – Summer 2008

Page 2 of 8

Closing Stock

Raw material (H) (Working 3) 48,342,000 Finished Goods (P) (Working 4) 77,508,667 Creditors - Raw material A (L x 30/360) (24,518,583) Raw material B (M x 45/360) (24,518,583) Labour, FOH and Admin Expenses (B x 0.38 x 15/360) (19,791,667) 136,933,382

Working 1 Sales in Previous year A 1,000,000,000 Sales in Current year ( A x 1.25 ) B 1,250,000,000 Local sales ( A x 60%x1.1x1.15 ) C 759,000,000 Exports (B-C) D 491,000,000 Working 2 Assume sale with 1% discount = X Sale with 2% discount will be = 2X Discount = 0.01X+(0.02*2X) = 0.05 X = Rs. 6,000,000 Therefore sale on which 1% discount will be given = X = 6,000,000/0.05 E 120,000,000 Therefore sale on which 2% discount will be given = 2X = 120,000,000*2 F 240,000,000 Working 3 Local sales at last year's price (1 billion * 60% * 1.1) 660,000,000 Exports as above 491,000,000 Total Sales excluding the effect of price increase G 1,151,000,000

Purchases of Raw Material Closing stock of Raw material (G*0.48*1.05/12) H 48,342,000 Raw material included in cost of sales (G*0.48*1.05) I 580,104,000 Opening stock of Raw material (A*0.48/12) J (40,000,000) Total raw material purchases K 588,446,000

Purchases of A (K*3/6) L 294,223,000 Purchases of B (K*2/6) M 196,148,667 Purchases of C (K*1/6) N 98,074,333 Working 4 Raw material as above (H) 48,342,000 Labour and factory overheads (B*28%/12) 29,166,667

P 77,508,667

MANAGEMENT ACCOUNTING Suggested Answer

Final Examinations – Summer 2008

Page 3 of 8

Ans.3

(a) (70% = Rs. 2.8 bn)

(A) Fav = 60% = -0.5 bn (30% = Rs. 0.8 bn) (Research = -0.1 bn) (20% = Rs. 0.7 bn)

(B) Unfav = 40% = -0.5 bn (80% = Rs. -0.4 bn) (40% = 2.0 bn) (No research = -0.5 bn) (35% = 1.2 bn) (C) (25% = -0.2 bn)

Rs.

billion

0.7

0.6 0.3 0.5 bln

0.2 0.4 0.8 0.4 0.35

0.5 bln 0.25

= Decision point (DP)

= Chance point (CP)

(i) At CP2, EV = (0.7 × 2.8) + (0.3 × 0.8) = 1.96 + 0.24 = 2.2 billion (ii) At CP3, EV = (0.2 × 0.7) + (0.8 × –0.4) = 0.14 – 0.32 = –0.18 billion (iii) At CP1, EV = (0.6 × 2.20) + (0.4 × –0.18) = 1.32 – 0.072 = 0.1248 billion (iv) At DP B, EV = 1.248 – 0.5 = 0.748 billion (v) At DP A, EV = 0.748 – 01 = 0.648 billion (vi) At CP4, EV = (0.4 × 2.0) + (0.35 × 1.20) – 0.25 × –0.2) = 0.8 + 0.42 – 0.05 = 1.17

billion (vii) At DP C and A, EV = 1.17 – 0.5 = 0.670 billion The company’s profits would be higher by Rs. 220 million (0.670 billion – 0.648 billion)

if it did not carry out research.

No research

Undertake research

0.1 billion

Initial launch

Initial launch

Feasible

Not feasible

2.8

0.8

0.7

-0.4

2.0

1.2

-0.2

MANAGEMENT ACCOUNTING Suggested Answer

Final Examinations – Summer 2008

Page 4 of 8

(b) Expected profit if the research is carried out Profit expectation at point A = (2.2 x 70% x 60%) + (0.2 x 30% x 60%) = 0.924+ 0.036 = 0.960 billion

Profit expectation at point B = (0.1 x 0.2 x 0.4) – (1.0 x 80% x 40%) = (0.008 - 0.32) = -0.312

Profit expectation if research is carried out = 0.960 – 0.312 = 0.648 bln

Expected profit if research is not carried out (Point C) (1.5 x 40%) + (0.7 x 35%) – (0.7 x 25%) = 0.6 + 0.245 – 0.175 = 0.67bn The company should not carry out research, as then it could earn higher profit of Rs. 22

million (670 – 648). Ans.3

(b) No. of units

Average hours

Total hours

1 5,000 5,000 2 4,000 8,000 4 3,200 12,800 8 2,560 20,480

16 2,048 32,768

Hours used for 9-16 units (32768-20480) 12,288

Cost of labour 12288 x 100 Rs. 1,228,800 Direct materials 8*400000 Rs. 3,200,000 Variable overheads 12288 x 80% Rs. 983,040 5,411,840 Margin 1,352,960 Sale price Rs. 6,764,800 Ans.4 Computation of Units Sold

Rupees

Actual Sales Price per unit (100 / 1.0526) 95 Sales price variance per unit (100 – 95) (A) 5 Adverse Selling Price Variance (B) 24,250,000 Units Sold during the period B / A 4,850,000 Computation of Units Manufactured Million units Units Sold 4.85 Increase in inventory level 0.23 Units Manufactured 5.08

MANAGEMENT ACCOUNTING Suggested Answer

Final Examinations – Summer 2008

Page 5 of 8

(a) Actual cost of raw materials consumed (million rupees)

Standard cost Price variance Mix

variance* Actual cost

X (5.08*15) 76.2000 2.2950 (7.6200) 70.8750 Y (5.08*20) 101.6000 (2.7030) 6.0960 104.9930 Z (5.08*27) 137.1600 (3.7995) (0.9140) 132.4465 314.9600 (4.2075) (2.4380) 308.3145 *See mix variance working (b) Material mix variance

Standard mix (millions Kgs)

Actual mix (million Kgs)

Difference (million Kgs)

Variance (million Rs.)

X (5.08*5) 25.40 22.860 2.540 7.620 Y (5.08*10) 50.80 53.848 -3.048 -6.096 Z (5.08*15) 76.20 75.692 0.508 0.914 152.40 152.400 2.438 (c) Labour Cost Variance

Quantity consumed

million Kgs

Actual labour at standard

cost

Standard labour cost

Labour cost variance

million Rs.

skilled 22.860 22.860 25.400 2.540 Fav semi-skilled 53.848 40.386 38.100 -2.286 Adv unskilled 75.692 7.569 7.620 0.051 Fav 152.400 71.094 71.400 0.306 Ans.5

Activity 1 2 3 4

Unallocated Total

(a) Manufactur-

ing Customer

service

Order process-

ing

Ware-housing

Indirect labour 4,320,000 1,440,000 - 1,440,000 - 7,200,000

Other manufacturing overheads 8,550,000 - - 450,000 - 9,000,000

Quality Control 900,000 600,000 - - - 1,500,000 Transportation 126,000 882,000 - 252,000 - 1,260,000

Admin salaries - - 600,000 750,000 1,650,000 3,000,000 13,896,000 2,922,000 600,000 2,892,000 1,650,000 21,960,000

Budgeted activity level 72,000 120 20 40,000,000

Cost driver rate 193.00 24,350.00 30,000.00 0.0723

per labour hour

per order day

per order processed

per Re. of material usage

MANAGEMENT ACCOUNTING Suggested Answer

Final Examinations – Summer 2008

Page 6 of 8

(b) Order by KSL Costs under present method Direct material cost 3,000,000 Direct labour 1,500,000 Factory overheads (90% of direct labour) 1,350,000 5,850,000 Mark-up - 50% 2,925,000 Sale price (A) 8,775,000 Costs under ABC Method Direct material cost 3,000,000 Direct labour 1,500,000 Other manufacturing cost (6,000 x 193) 1,158,000 Customer service (10 x 24,350) 243,500 Order processing (1 x 30,000) 30,000 Warehousing (3,000,000 x 0.0723) 216,900 6,148,400 Margin -20% of sales price 1,537,100 Sale price (B) 7,685,500 Discount that may be allowed (A-B) 1,089,500

Ans.6 (a) EARNINGS UNDER PROPOSED OPTION Total billing to customers (working 1) 9,000,000 Less: Cost of raw material used after warranty period (working 3) 2,400,000 Cost of labour & variable overhead (3.8 mln + 10%) (working 2) 4,180,000 Salary of Supervisor 480,000 Increase in other fixed overheads 720,000 7,780,000 Net profit excluding cost of material used during warranty period 1,220,000 LESS: EARNINGS UNDER THE PRESENT OPTION Mark-up earned on supply of material 360,000 Share of billing received from AHA 990,000 1,350,000

Less: Payment to AHA for services provided during warranty period (760,000+30%) (working 2) 988,000

362,000 Net savings 858,000

Working 1

Domestic

Customers Industrial Customers

Total

Ratio of services provided by AHA A 20 80 100 Share of KL in % B 15.00% 10.00% N/A Ratio of KL's share C (A*B) 3 8 11

MANAGEMENT ACCOUNTING Suggested Answer

Final Examinations – Summer 2008

Page 7 of 8

Annual share received from AHA D 270,000 720,000 990,000 Total billing by AHA E (D/B) 1,800,000 7,200,000 9,000,000

Working 2

Net recoveries from customers F (E-D) 1,530,000 6,480,000 8,010,000

Less: Recoveries in respect of material (See working 3) G 3,450,000

Recoveries in respect of services (labour & overheads) H (F-G) 4,560,000

Cost of labour and overhead incurred by AHA (after warranty period) J (H*100/150) 3,040,000

Cost of labour and overhead incurred by AHA (during warranty period) K (J*20/80) 760,000

Total cost of labour and overhead L(J + K) 3,800,000 Working 3 Mark-up charged by KL on material billed to AHA 360,000 Cost of material despatched (for use after warranty period) 2,400,000 2,760,000 Mark-up charged by AHA on material billed to customers (2,760,000*0.25) 690,000 Total billing in respect of material 3,450,000 Alternative Answer 6(a) Savings/additional revenues if services provided by KL

Mark-up charged by AHA, from the customers on cost of material (working 1) 690,000

Mark-up charged by AHA from KL on services provided during warranty period

L (K*0.3) (working 2) 228,000

Mark-up charged by AHA, from the customers on cost of labour and overhead

M (H-J) (working 2) 1,520,000

Total 2,438,000 Less: Additional costs and decline in revenues

Increase in cost of labour and variable overheads N ((J+K)*0.1)

(working 2) 380,000

Supervisor’s salary 480,000 Increase in other fixed overheads 720,000 1,580,000 Net savings 858,000 Working 1 Mark-up charged by KL on material billed to AHA 360,000 Cost of material ispatched (for use after warranty period) 2,400,000 2,760,000 Mark-up charged by AHA on material billed to customers 2760000*0.25 690,000 Total billing in respect of material 3,450,000

MANAGEMENT ACCOUNTING Suggested Answer

Final Examinations – Summer 2008

Page 8 of 8

Working 2

Domestic

Customers Industrial Customers

Total

Ratio of services provided by AHA A 20 80 100 Share of KL in % B 15.00% 10.00% N/A Ratio of KL’s share C (A*B) 3 8 11 Annual share received from AHA D 270,000 720,000 990,000 Total billing by AHA E (D/B) 1,800,000 7,200,000 9,000,000 Net recoveries from customers F (E-D) 1,530,000 6,480,000 8,010,000 Less: Recoveries in respect of material G (see working 1) 3,450,000

Recoveries in respect of services (labour & overheads) H (F-G) 4,560,000

Cost of labour and overhead incurred by AHA (after warranty period) J (H*100/150) 3,040,000

Cost of labour and overhead incurred by AHA (during warranty period) K (J*20/80) 760,000

Total cost of materials L(J + K) 3,800,000

(b) (i) It might be beneficial for Kamran Limited (KL) to focus on core business rather than on non-core areas like after-sale service.

(ii) Ahmed Hasan Associates (AHA) might be technically more competent at providing these services.

(iii) KL should also consider the reliability of AHA as an outside supplier of these services. If after-sale service is a critical component of KL’s business, it might be better to do it in-house.

(iv) There is a potential for KL to be inefficient in terms of cost control during parts production since the company charges a cost-plus margin to AHA. There is not much incentive for KL to control costs.

(v) The numbers provided by the cost accountant might be misleading since these are predominantly direct costs of providing the service and possible effects on other overheads may not have been considered.

(THE END)